NUR 205- MedSurg 2- Test 2

Pataasin ang iyong marka sa homework at exams ngayon gamit ang Quizwiz!

As part of the process of checking the placement of a nasogastric tube, the nurse checks the pH of the aspirate. Which pH finding would indicate to the nurse that the tube is in the stomach? 1- 4 2- 6 3- 8 4- 10

1

Which statement is correct regarding glargine insulin? 1- Its peak action occurs in 2 to 3 hours. 2- It cannot be mixed with any other type of insulin. 3- It is absorbed rapidly. 4- It is given twice daily.

2

A client with diabetes is receiving an oral anti diabetic agent that acts to help the tissues use available insulin more efficiently. Which of the following agents would the nurse expect to administer? 1- Metformin 2- Glyburide 3- Repaglinide 4- Glipizide

1

Rebound hypoglycemia is a complication of parenteral nutrition caused by 1- glucose intolerance. 2- fluid infusing rapidly. 3- feedings stopped too abruptly. 4- a cap missing from the port.

3

A client who is 24 hours post op from laparoscopic cholecystectomy calls the nurse and reports pain in the right shoulder. How should the nurse respond to the client's report of symptoms? "Apply a heating pad to your shoulder for 15 minutes hourly as needed." "Come into the emergency room as soon as possible." "Take an over the counter analgesic as needed." "Place your shoulder in a sling to avoid moving it."

"Apply a heating pad to your shoulder for 15 minutes hourly as needed." Explanation: Pain in the right shoulder may occur after laparoscopic cholecystectomy due to migration of the carbon dioxide used to insufflate the abdominal cavity during the procedure. The nurse should instruct the client to apply a heating pad to the shoulder for 15 to 20 minutes every hour as needed for pain relief. The nurse should not instruct the client to take analgesic medication-this is a medical order performed by the health care provider only. This scenario is not life threatening and the client does not need to go to the emergency department. It is also not necessary for the client to place the shoulder in a sling as this is not an injury-related condition.

A home care nurse is caring for a client with reports of epigastric discomfort who is scheduled for a barium swallow. Which statement by the client indicates an understanding of the test? "I'll avoid eating or drinking anything 6 to 8 hours before the test." "I'll drink full liquids the day before the test." "There is no need for special preparation before the test." "I'll take a laxative to clear my bowels before the test."

"I'll avoid eating or drinking anything 6 to 8 hours before the test." Explanation: The client demonstrates understanding of a barium swallow when stating he or she must refrain from eating or drinking for 6 to 8 hours before the test. No other preparation is needed. Before a lower GI series, the client should eat a low-residue or clear liquid diet for 2 days and take a potent laxative and an oral liquid preparation.

A client asks the nurse why the physician ordered the blood test carcinoembryonic antigen (CEA). The nurse answers: "It tells the physician what type of cancer is present." "It indicates if a cancer is present." "It determines functionality of the liver." "It detects a protein normally found in the blood."

"It indicates if a cancer is present." Explanation: The carcinoembryonic antigen (CEA) blood test detects the presence of cancer by identifying the presence of a protein not normally detected in the blood of a healthy person. However, it does not indicate what type of cancer is present nor does it detect the functionality of the liver.

A student accepted into a nursing program must begin receiving the hepatitis B series of injections. The student asks when the next two injections should be administered. What is the best response by the instructor? "You must have the second one in 2 weeks and the third in 1 month." "You must have the second one in 1 month and the third in 6 months." "You must have the second one in 6 months and the third in 1 year." "You must have the second one in 1 year and the third the following year."

"You must have the second one in 1 month and the third in 6 months." Explanation: Both forms of the hepatitis B vaccine are administered intramuscularly in three doses; the second and third doses are given 1 and 6 months, respectively, after the first dose.

A client presented with gastrointestinal bleeding 2 days ago and continues to have problems. The health care provider has ordered a visualization of the small intestine via a capsule endoscopy. What will the nurse include in the client education about this procedure? "A capsule will be inserted into your rectum." "You will need to swallow a capsule." "The health care provider will use a scope called a capsule to view your intestine." "An x-ray machine will use a capsule ray to follow your intestinal tract."

"You will need to swallow a capsule." Explanation: A capsule endoscopy allows for noninvasive visualization of the small intestinal mucosa. The technique consists of the client swallowing a capsule that is embedded with a wireless miniature camera, which is propelled through the intestine by peristalsis. The capsule passes from the rectum in 1 to 2 days.

A client has been receiving radiation therapy to the lungs and now has erythema, edema, and pain of the mouth. What instruction will the nurse give to the client? 1- Use a hard-bristled toothbrush. 2- Rinse with an alcohol-based solution. 3- Brush and floss daily. 4- Continue with the usual diet.

3

The nurse is to discontinue a nasogastric tube that had been used for decompression. What is the first action the nurse should take? 1- Remove the tape from the nose of the client. 2- Withdraw the tube gently for 6 to 8 inches. 3- Provide oral hygiene. 4- Flush with 10 mL of water.

4

The nurse practitioner worked with a registered dietitian to customize a 1,800-calorie diabetic diet for a 53-year-old man with special dietary needs. Which of the following percent distribution of calories should be provided? 1- Carbohydrates 35%, fat 10%, and protein 5% 2- Carbohydrates 40%, fat 15%, and protein 10% 3- Carbohydrates 45%, fat 20%, and protein 15% 4- Carbohydrates 50%, fat 25%, and protein 20%

4

The nurse provides health teaching to inform the client with oral cancer that 1- most oral cancers are painful at the outset. 2- blood testing is used to diagnose oral cancer. 3- a typical lesion is soft and craterlike. 4- many oral cancers produce no symptoms in the early stages.

4

A patient taking corticosteroids for exacerbation of Crohn's disease comes to the clinic and informs the nurse that he wants to stop taking them because of the increase in acne and moon face. What can the nurse educate the patient regarding these symptoms? A The moon face and acne will resolve when the medication is tapered off. B Those symptoms are not related to the corticosteroid therapy. C The dose of the medication must be too high and should be lowered. D The symptoms are permanent side effects of the corticosteroid therapy.

A

The nurse is closely monitoring the blood work of a patient who has a diagnosis of primary hyperparathyroidism. The nurse should be aware that the fluid and electrolyte disturbances associated with this disease create a significant risk of what problems? A Renal calculi and urinary obstruction B Metabolic acidosis and cardiac ischemia C Fluid volume overload and pruritus D Deep vein thrombosis and pulmonary embolism

A

The nurse on the telemetry floor is caring for a patient with long-standing hypothyroidism who has been taking synthetic thyroid hormone replacement sporadically. What is a priority that the nurse monitors for in this patient? A Symptoms of acute coronary syndrome B Symptoms of pneumonia C Dietary intake of foods with saturated fats D Heat intolerance

A

Which of the following is a clinical manifestation of hypothyroidism? A A pulse rate below 60 beats/minute. B Systolic murmurs C Exophthalmos D An elevated systolic blood pressure.

A

A client receiving thyroid replacement therapy develops influenza and forgets to take her thyroid replacement medicine. The nurse understands that skipping this medication puts the client at risk for developing which life-threatening complication? A Myxedema coma B Thyroid storm C Tibial myxedema D Exophthalmos

A Myxedema coma, severe hypothyroidism, is a life-threatening condition that may develop if thyroid replacement medication isn't taken. Exophthalmos (protrusion of the eyeballs) is seen with hyperthyroidism. Although thyroid storm is life-threatening, it's caused by severe hyperthyroidism.

A client is suspected of having cirrhosis of the liver. What diagnostic procedure will the nurse prepare the client for in order to obtain a confirmed diagnosis? A liver biopsy A CT scan A prothrombin time Platelet count

A liver biopsy Explanation: A liver biopsy, which reveals hepatic fibrosis, is the most conclusive diagnostic procedure. It can be performed in the radiology department with ultrasound or CT to identify appropriate placement of the trocar or biopsy needle. A prothrombin time and platelet count will assist with determining if the client is at increased risk for bleeding

The nurse reviews the laboratory values for a patient being evaluated for alcoholic cirrhosis. The nurse is aware that a diagnostic indicator present in greater than 70% of cases is which of the following? Total bilirubin AST/ALT ratio >3.0 Alkaline phosphatase Albumin

AST/ALT ratio >3.0 Explanation: The aspartate aminotransferase/alanine aminotransferase (AST/ALT) ratio is helpful in diagnosing liver damage and is present in more than 70% of cases.

Which of the following is the primary function of the small intestine? Absorption Digestion Peristalsis Secretion

Absorption Explanation: Absorption is the primary function of the small intestine. Digestion occurs in the stomach. Peristalsis occurs in the colon. The duodenum secretes enzymes.

An important message for any nurse to communicate is that drug-induced hepatitis is a major cause of acute liver failure. The medication that is the leading cause is: Acetaminophen Ibuprofen Dextromethorphan Benadryl

Acetaminophen Explanation: Although any medication can affect liver function, use of acetaminophen (found in many over-the-counter medications used to treat fever and pain) has been identified as the leading cause of acute liver failure. Other medications commonly associated with liver injury include anesthetic agents, medications used to treat rheumatic and musculoskeletal disease, antidepressants, psychotropic medications, anticonvulsants, and antituberculosis agents. MAX YOU CAN TAKE PER DAY IS 4,000 MG

A client with a history of alcohol abuse comes to the emergency department and complains of abdominal pain. Laboratory studies help confirm a diagnosis of acute pancreatitis. The client's vital signs are stable, but the client's pain is worsening and radiating to his back. Which intervention takes priority for this client? Placing the client in a semi-Fowler's position Maintaining nothing-by-mouth (NPO) status Administering morphine I.V. as ordered Providing mouth care

Administering morphine I.V. as ordered Explanation: The nurse should address the client's pain issues first by administering morphine I.V. as ordered. Placing the client in a Semi-Fowler's position, maintaining NPO status, and providing mouth care don't take priority over addressing the client's pain issues.

The nurse is planning care for a client following an incisional cholecystectomy for cholelithiasis. Which intervention is the highestnursing priority for this client? Assisting the client to turn, cough, and deep breathe every 2 hours Teaching the client to choose low-fat foods from the menu Performing range-of-motion (ROM) leg exercises hourly while the client is awake Assisting the client to ambulate the evening of the operative day

Assisting the client to turn, cough, and deep breathe every 2 hours Explanation: Assessment should focus on the client's respiratory status. If a traditional surgical approach is planned, the high abdominal incision required during surgery may interfere with full respiratory excursion. The other nursing actions are also important, but are not as high a priority as ensuring adequate ventilation.

Which of the following would the nurse expect to assess in a client with hepatic encephalopathy? Increased motor activity Asterixis Negative Babinski reflex Irritability

Asterixis Explanation: Hepatic encephalopathy is manifested by numerous central nervous system effects including: disorientation, confusion, personality changes, memory loss, a flapping tremor called asterixis, a positive Babinski reflex, sulfurous breath odor (referred to as fetor hepaticus), and lethargy to deep coma.

The nurse is caring for a patient with hepatic encephalopathy. While making the initial shift assessment, the nurse notes that the patient has a flapping tremor of the hands. What will the nurse document this condition as in the patient's chart? Asterixis Constructional apraxia Fetor hepaticus Palmar erythema

Asterixis Explanation: The nurse will document that a patient exhibiting a flapping tremor of the hands is demonstrating asterixis. Constructional apraxia is also a motor disturbance, but it is the inability to reproduce a simple figure. Fetor hepaticus is a sweet, slightly fecal odor to the breath and not associated with a motor disturbance. Skin changes associated with liver dysfunction may include palmar erythema, which is a reddening of the palms, but is not a flapping tremor.

A client with severe hypoparathyroidism is experiencing tetany. What medication, prescribed by the physician for emergency use, will the nurse administer to correct the deficit? A. Fludrocortisone B. Calcium gluconate C Methylprednisolone D Sodium bicarbonate

B

A nurse is caring for a client with Cushing's syndrome. Which would the nurse not include in this client's plan of care? A. Administer prescribed diuretics. B. Provide a high-sodium diet. C. Report systolic BP that exceeds 139 mm Hg or diastolic BP that exceeds 89 mm Hg. D. examine extremities for pitting edema.

B

A nurse is preparing an IV dose of hydrocortisone that is to be administered to an adult patient on an acute medical unit. The endocrine disorder for which this treatment is most clearly indicated is: A. Cushing's syndrome B Addison's disease C Diabetes insipidus D Syndrome of inappropriate antidiuretic hormone (SIADH)

B

A 47-year-old man with epigastric pain is being admitted to the hospital. During the admission assessment and interview, what specific information should the nurse obtain from the patient, who is suspected of having peptic ulcer disease? A. Any allergies to food or medications B. Use of nonsteroidal anti-inflammatory drugs (NSAIDs) C. Medical history for two previous generations D. History of side effects of all medications

B. Use of nonsteroidal anti-inflammatory drugs (NSAIDs) RATIONALE Use of NSAIDs in the patient suspected of peptic ulcer disease increases the risk of GI bleeding.

A client who had developed jaundice 2 months earlier is brought to the ED after attending a party and developing excruciating pain that radiated over the abdomen and into the back. Upon assessment, which additional symptom would the nurse expect this client to have? Hypertension Bile-stained vomiting Warm, dry skin Weight loss

Bile-stained vomiting Explanation: Nausea and vomiting are common in acute pancreatitis. The emesis is usually gastric in origin but may also be bile stained. Fever, jaundice, mental confusion, and agitation may also occur.

The nurse is assessing a client for acromegaly at the clinic. Besides asking about changes in shoe size and facial features, the nurse should also inquire about changes in which of the following? A. Hearing B. Bowel habits C. Vision D. Taste of foods

C. Vision RATIONALE Oversecretion of the growth hormone from the anterior pituitary results in acromegaly. The pituitary gland may be enlarged and causing a pressure on the optic nerve in the brain, thus changes in vision may occur.

When caring for a client with advanced cirrhosis and hepatic encephalopathy, which assessment finding should the nurse report immediately? Weight loss of 2 pounds in 3 days Change in the client's handwriting and/or cognitive performance Anorexia for more than 3 days Constipation for more than 2 days

Change in the client's handwriting and/or cognitive performance Explanation: The earliest symptoms of hepatic encephalopathy include mental status changes and motor disturbances. The client will appear confused and unkempt and have altered mood and sleep patterns. Neurologic status should be assessed frequently. Mental status is monitored by the nurse keeping the client's daily record of handwriting and arithmetic performance. The nurse should report any change in mental status immediately. Chronic fatigue, anorexia, dyspepsia, nausea, vomiting, and diarrhea or constipation with accompanying weight loss are regular symptoms of cirrhosis.

Which is a clinical manifestation of cholelithiasis? Epigastric distress before a meal Clay-colored stools Abdominal pain in the upper left quadrant Nonpalpable abdominal mass

Clay-colored stools Explanation: The client with gallstones has clay-colored stools and excruciating upper right quadrant pain that radiates to the back or right shoulder. The excretion of bile pigments by the kidneys makes urine very dark. The feces, no longer colored with bile pigments, are grayish (like putty) or clay colored. The client develops a fever and may have a palpable abdominal mass.

A client who is being treated for pyloric obstruction has a nasogastric (NG) tube in place to decompress the stomach. The nurse routinely checks for obstruction which would be indicated by what amount? A. 150 mL B. 250 mL C. 350 mL D. 450 mL

D. 450 mL RATIONALE A residual of greater than 400 mL strongly suggests obstruction.

A nurse is caring for a client who underwent a subtotal gastrectomy. To manage dumping syndrome, the nurse should advise the client to: A. restrict fluid intake to 1 qt (1,000 ml)/day. B. drink liquids only with meals. C. don't drink liquids 2 hours before meals. D. drink liquids only between meals.

D. drink liquids only between meals. RATIONALE A client who experiences dumping syndrome after a subtotal gastrectomy should be advised to ingest liquids between meals rather than with meals. Taking fluids between meals allows for adequate hydration, reduces the amount of bulk ingested with meals, and aids in the prevention of rapid gastric emptying. There is no need to restrict the amount of fluids, just the time when the client drinks fluids. Drinking liquids with meals increases the risk of dumping syndrome by increasing the amount of bulk and stimulating rapid gastric emptying. Small amounts of water are allowable before meals.

Increased appetite and thirst may indicate that a client with chronic pancreatitis has developed diabetes mellitus. Which of the following explains the cause of this secondary diabetes? Dysfunction of the pancreatic islet cells Ingestion of foods high in sugar Inability for the liver to reabsorb serum glucose Renal failure

Dysfunction of the pancreatic islet cells Explanation: Diabetes mellitus resulting from dysfunction of the pancreatic islet cells is treated with diet, insulin, or oral antidiabetic agents. The hazard of severe hypoglycemia with alcohol consumption is stressed to the client and family. When secondary diabetes develops in a client with chronic pancreatitis, the client experiences increased appetite, thirst, and urination. A standard treatment with pancreatitis is to make the client NPO. The dysfunction is related to the pancreas, not the liver.

What test should the nurse prepare the client for that will locate stones that have collected in the common bile duct? Colonoscopy Abdominal x-ray Cholecystectomy Endoscopic retrograde cholangiopancreatography (ERCP)

Endoscopic retrograde cholangiopancreatography (ERCP) Explanation: ERCP locates stones that have collected in the common bile duct. A colonoscopy will not locate gallstones but only allows visualization of the large intestine. Abdominal x-ray is not a reliable locator of gallstones. A cholecystectomy is the surgical removal of the gallbladder. Reference:

A group of students is reviewing information about the liver and associated disorders. The group demonstrates understanding of the information when they identify which of the following as a primary function of the liver? Breakdown amino acids Convert urea into ammonia Excrete bile Break down coagulation factors

Excrete bile Explanation: The liver forms and excretes bile, synthesizes amino acids from the breakdown of proteins, converts ammonia into urea, and synthesizes the factors needed for blood coagulation.

Which type of deficiency results in macrocytic anemia? Folic acid Vitamin C Vitamin A Vitamin K

Folic acid Explanation: Folic acid deficiency results in macrocytic anemia. Vitamin C deficiency results in hemorrhagic lesions of scurvy. Vitamin A deficiency results in night blindness and eye and skin changes. Vitamin K deficiency results in hypoprothrombinemia, which is characterized by spontaneous bleeding and ecchymosis.

Which condition in a client with pancreatitis makes it necessary for the nurse to check fluid intake and output, check hourly urine output, and monitor electrolyte levels? Frequent vomiting, leading to loss of fluid volume Dry mouth, which makes the client thirsty Acetone in the urine High glucose concentration in the blood

Frequent vomiting, leading to loss of fluid volume Explanation: Fluid and electrolyte disturbances are common complications because of nausea, vomiting, movement of fluid from the vascular compartment to the peritoneal cavity, diaphoresis, fever, and the use of gastric suction. The nurse assesses the client's fluid and electrolyte status by noting skin turgor and moistness of mucous membranes. The nurse weighs the client daily and carefully measures fluid intake and output, including urine output, nasogastric secretions, and diarrhea.

Which type of jaundice seen in adults is the result of increased destruction of red blood cells? Hemolytic Hepatocellular Obstructive Nonobstructive

Hemolytic Explanation: Hemolytic jaundice results because, although the liver is functioning normally, it cannot excrete the bilirubin as quickly as it is formed. Obstructive and hepatocellular jaundice are results of liver disease. Nonobstructive jaundice occurs with hepatitis.

A 33-year-old male patient with a history of IV heroin and cocaine use has been admitted to the medical unit for the treatment of endocarditis. The nurse should recognize that this patient is also likely to test positive for which of the following hepatitis viruses? Hepatitis A Hepatitis B Hepatitis C Hepatitis D

Hepatitis C Explanation: Transmission of hepatitis C occurs primarily through injection of drugs and through transfusion of blood products prior to 1992. Hepatitis A, B, and D are less likely to result from IV drug use.

A nurse is preparing a presentation for a local community group about hepatitis. Which of the following would the nurse include? Hepatitis B is transmitted primarily by the oral-fecal route. Hepatitis A is frequently spread by sexual contact. Hepatitis C increases a person's risk for liver cancer. Infection with hepatitis G is similar to hepatitis A.

Hepatitis C increases a person's risk for liver cancer. Explanation: Infection with hepatitis C increases the risk of a person developing hepatic (liver) cancer. Hepatitis A is transmitted primarily by the oral-fecal route; hepatitis B is frequently spread by sexual contact and infected blood. Hepatitis E is similar to hepatitis A whereas hepatitis G is similar to hepatitis C.

Gastrin has which of the following effects on gastrointestinal (GI) motility? Increased motility of the stomach Relaxation of the colon Contraction of the ileocecal sphincter Relaxation of gastroesophageal sphincter

Increased motility of the stomach Explanation: Gastrin has the following effects on GI motility: increased motility of the stomach, excitation of the colon, relaxation of ileocecal sphincter, and contraction of the gastroesophageal sphincter.

When examining the abdomen of a client with reports of nausea and vomiting, what would the nurse do first? Palpation Inspection Auscultation Percussion

Inspection Explanation: When assessing the abdomen, the nurse would first inspect or observe the abdomen. This would be followed by auscultation, percussion, and lastly, palpation.

While conducting a physical examination of a client, which of the following skin findings would alert the nurse to the liklihood of liver problems? Select all that apply. Jaundice Petechiae Ecchymoses Cyanosis of the lips Aphthous stomatitis

Jaundice Petechiae Ecchymoses Explanation: The skin, mucosa, and sclerae are inspected for jaundice. The nurse observes the skin for petechiae or ecchymotic areas (bruises), spider angiomas, and palmar erythema. Cyanosis of the lips is indicative of a problem with respiratory or cardiovascular dysfunction. Aphthous stomatitis is a term for mouth ulcers and is a gastrointestinal abnormal finding.

Clinical manifestations of common bile duct obstruction include all of the following except: Light-colored urine Clay-colored feces Pruritus Jaundice

Light-colored urine Explanation: The excretion of the bile pigments by the kidneys gives the urine a very dark color. The feces, no longer colored with bile pigments, are grayish, like putty, or clay-colored. The symptoms may be acute or chronic. Epigastric distress, such as fullness, abdominal distention, and vague pain in the right upper quadrant of the abdomen, may occur. If it goes untreated jaundice and pruritus can occur.

After 20 seconds of auscultating for bowel sounds on a client recovering from abdominal surgery, the nurse hears nothing. What should the nurse do based on the assessment findings? Listen longer for the sounds. Document that the client is constipated. Call the health care provider to report absent bowel sounds. Return in 1 hour and listen again to confirm findings.

Listen longer for the sounds. Explanation: Auscultation is used to determine the character, location, and frequency of bowel sounds. The frequency and character of sounds are usually heard as clicks and gurgles that occur irregularly and range from 5 to 35 per minutes. Normal sounds are heard about every 5 to 20 seconds, whereas hypoactive sounds can be one or two sounds in 2 minutes. Postoperatively, it is common for sounds to be reduced; therefore, the nurse needs to listen at least 3 to 5 minutes to verify absent or no bowel sounds.

The nurse is caring for a geriatric client and notices polypharmacy. Which diagnostic studies are anticipated? Complete blood count Urinalysis Liver function studies Blood chemistry

Liver function studies Explanation: The liver metabolizes and biotransforms the medications ingested. Geriatric clients who experience polypharmacy or multiple medications have an elevated risk of liver impairment. Routine liver function studies monitor the status of the liver and its ability to metabolize.

A 59-year-old male patient was diagnosed with cirrhosis several years ago. After years of successful management of his chronic disease, he experienced a constellation of symptoms that eventually resulted in a diagnosis of hepatocellular carcinoma (HCC). What treatment holds the greatest potential for curing this patient's health problem? Hepatic lobectomy Radiotherapy Liver transplantation Laser ablation

Liver transplantation Explanation: In patients with HCC and cirrhosis, liver transplantation is the treatment of choice, being deemed more effective than surgery and radiotherapy.

When performing a physical examination on a client with cirrhosis, a nurse notices that the client's abdomen is enlarged. Which of the following interventions should the nurse consider? Report the condition to the physician immediately. Measure abdominal girth according to a set routine. Provide the client with nonprescription laxatives. Ask the client about food intake.

Measure abdominal girth according to a set routine. Explanation: If the abdomen appears enlarged, the nurse measures it according to a set routine. The nurse reports any change in mental status or signs of gastrointestinal bleeding immediately. It is not essential for the client to take laxatives unless prescribed. The client's food intake does not affect the size of the abdomen in case of cirrhosis.

A client with acute pancreatitis has been started on total parenteral nutrition (TPN). Which action should the nurse perform after administration of the TPN? Auscultate the abdomen for bowel sounds every 4 hours Measure abdominal girth every shift Monitor for reports of nausea and vomiting Measure blood glucose concentration every 4 to 6 hours

Measure blood glucose concentration every 4 to 6 hours Explanation: Enteral or parenteral nutrition may be prescribed. In addition to administering enteral or parenteral nutrition, the nurse monitors the serum glucose concentration every 4 to 6 hours.

The critical care nurse is caring for a patient with cirrhosis. What is a priority nursing function when caring for a patient with cirrhosis? Monitoring the patient's oral intake Monitoring the patient's social support network Monitoring the patient for signs of hypervolemia Monitoring the patient's mental status

Monitoring the patient's mental status Explanation: Monitoring is an essential nursing function to identify early deterioration in mental status. The nurse monitors the patient's mental status closely and reports changes so that treatment of encephalopathy can be initiated promptly. An extensive neurologic evaluation is key to identifying progression through the four stages of encephalopathy. The nurse would monitor the oral intake and watch for signs of hypervolemia, but they are not as essential as the patient's mental status because of the encephalopathy that goes with cirrhosis. Monitoring the support network is not essential at this time.

A client has a blockage of the passage of bile from a stone in the common bile duct. What type of jaundice does the nurse suspect this client has? Hemolytic jaundice Hepatocellular jaundice Obstructive jaundice Cirrhosis of the liver

Obstructive jaundice Explanation: Obstructive jaundice is caused by a block in the passage of bile between the liver and intestinal tract. Hemolytic jaundice is caused by excess destruction of red blood cells. Hepatocellular jaundice is caused by liver disease. Cirrhosis of the liver would be an example of hepatocellular jaundice.

A client has undergone a liver biopsy. Which postprocedure position is appropriate? On the left side Trendelenburg On the right side High Fowler's

On the right side Explanation: In this position, the liver capsule at the site of penetration is compressed against the chest wall, and the escape of blood or bile through the perforation made for the biopsy is impeded. Positioning the client on his left side is not indicated. Positioning the client in the Trendelenburg position may be indicated if the client is in shock, but is not the position designed for the client after liver biopsy. The high Fowler position is not indicated for the client after liver biopsy.

Which of the following conditions is most likely to involve a nursing diagnosis of fluid volume deficit? Appendicitis Pancreatitis Cholecystitis Peptic ulcer

Pancreatitis Explanation: Hypotension is typical and reflects hypovolemia and shock caused by the loss of large amounts of protein-rich fluid into the tissues and peritoneal cavity. The other conditions are less likely to exhibit fluid volume deficit.

After being in remission from Hodgkin's disease for 18 months, a client develops a fever of unknown origin. A healthcare provider orders a liver biopsy to rule out advancing Hodgkin's disease and infection. Twenty-four hours after the biopsy, the client has a fever, complains of severe abdominal pain, and seems increasingly confused. What should the nurse suspect? Peritonitis from bleeding in the liver caused by the liver biopsy Perforation of the colon caused by the liver biopsy An allergic reaction to the contrast media used during the liver biopsy Normal postprocedural pain, with a change in the level of consciousness resulting from the pre-existing fever

Peritonitis from bleeding in the liver caused by the liver biopsy Explanation: After any invasive procedure, the nurse must stay alert for complications in the affected region—in this case, the abdomen. This client exhibits classic signs and symptoms of peritonitis caused by blood or bile after the liver biopsy. There is a reason to suspect bleeding resulting from the liver biopsy. It is rare to have a perforation of the colon after a biopsy. Liver biopsy doesn't involve the use of contrast media. The client's symptoms are not normal for a liver biopsy.

Following ingestion of carrots or beets, the nurse would expect which alteration in stool color? Red Black Yellow Milky white

Red Explanation: Carrots or beets will tend to change the stool color to red. Black stools are associated with iron, licorice, and charcoal. Senna is associated with yellow stools. A milky white stool is associated with administration of barium.

A client with cirrhosis has portal hypertension, which is causing esophageal varices. What is the goal of the interventions that the nurse will provide? Cure the cirrhosis. Treat the esophageal varices. Reduce fluid accumulation and venous pressure. Promote optimal neurologic function.

Reduce fluid accumulation and venous pressure. Explanation: Methods of treating portal hypertension aim to reduce fluid accumulation and venous pressure. There is no cure for cirrhosis; treating the esophageal varices is only a small portion of the overall objective. Promoting optimal neurologic function will not reduce portal hypertension.

A client is admitted to the health care facility with abdominal pain, a low-grade fever, abdominal distention, and weight loss. The physician diagnoses acute pancreatitis. What is the primary goal of nursing care for this client? Relieving abdominal pain Preventing fluid volume overload Maintaining adequate nutritional status Teaching about the disease and its treatment

Relieving abdominal pain Explanation: The predominant clinical feature of acute pancreatitis is abdominal pain, which usually reaches peak intensity several hours after onset of the illness. Therefore, relieving abdominal pain is the nurse's primary goal. Because acute pancreatitis causes nausea and vomiting, the nurse should try to prevent fluid volume deficit, not overload. The nurse can't help the client achieve adequate nutrition or understand the disease and its treatment until the client is comfortable and no longer in pain.

The nurse is providing care for a patient whose cancer has metastasized to her small intestine. What does the small intestine do? Select all that apply. Creation of human waste products Reabsorption of water to maintain blood pressure Secretion Absorption Movement of nutrients into the blood stream.

Secretion Absorption Movement of nutrients into the blood stream. Explanation: The small intestine is the longest segment of the gastrointestinal tract, accounting for about two-thirds of the total length. It folds back and forth on itself, providing approximately 7000 cm2 (70 m2) of surface area for secretion and absorption, the process by which nutrients enter the bloodstream through the intestinal walls. The colon makes the stool that is human waste. The small intestine does not reabsorb water to maintain blood pressure; this is a function of the kidneys.

Gynecomastia is a common side effect of which of the following diuretics? Spironolactone Furosemide Vasopressin Nitroglycerin (IV)

Spironolactone Explanation: Gynecomastia is a common side effect caused by spironolactone. Vasopressin is used for bleeding esophageal varices and is not a diuretic. Nitroglycerin (IV) may be used with vasopressin to counteract the effects of vasoconstriction from the vasopressin.

A client receives a local anesthetic to suppress the gag reflex for a diagnostic procedure of the upper GI tract. Which nursing intervention is advised for this patient? The client should not be given any food and fluids until the gag reflex returns. The client should be monitored for any breathing-related disorder or discomforts. The client's fluid output should be measured for at least 24 hours after the procedure. The client should be monitored for cramping or abdominal distention.

The client should not be given any food and fluids until the gag reflex returns. Explanation: For a client receiving a local anesthetic that suppresses the gag reflex, the nurse is advised to withhold food and fluids until the reflex returns.

An individual has had a snack consisting of half a bagel with cream cheese, lox (smoked salmon), red onions, and capers. Stimulation of the person's gastrointestinal tract has resulted in the secretion of numerous digestive enzymes into the small intestine, including trypsin. What component of this person's snack will be primarily digested by the action of trypsin? The bagel The lox The cream cheese The red onions and capers

The lox Explanation: Trypsin aids in digesting protein, such as fish. Amylase aids in digesting starch, such as the carbohydrates in a bagel. Lipase aids in digesting fats, such as those found in many dairy products.

The nurse is conducting an abdominal assessment of a patient who is postoperative day 1 following an open cholecystectomy. During auscultation of the patient's abdomen, the nurse has noted that clicks and gurgles are audible approximately every 10 seconds. How should the nurse follow up this assessment finding? The nurse should administer a p.r.n. stool softener. The nurse should contact the patient's care provider. The nurse should assess the patient for paralytic ileus. The nurse should document normoactive bowel sounds.

The nurse should document normoactive bowel sounds. Explanation: The frequency and character of bowel sounds are usually heard as clicks and gurgles that occur irregularly and range from 5 to 35 per minute. Bowel sounds occurring every 10 seconds would be an expected assessment finding that does not indicate the need for intervention.

A patient with acute pancreatitis puts the call bell on to tell the nurse about an increase in pain. The nurse observes the patient guarding; the abdomen is board-like and no bowel sounds are detected. What is the major concern for this patient? The patient requires more pain medication. The patient is developing a paralytic ileus. The patient has developed peritonitis. The patient has developed renal failure.

The patient has developed peritonitis. Explanation: Abdominal guarding is present. A rigid or board-like abdomen may develop and is generally an ominous sign, usually indicating peritonitis (Privette et al., 2011).

A client is to have an upper GI procedure with barium ingestion and abdominal ultrasonography. While scheduling these diagnostic tests, the nurse must consider which factor? Both tests need to be done before breakfast. The ultrasonography should be scheduled before the GI procedure. The upper GI should be scheduled before the ultrasonography. The client may eat a light meal before either test.

The ultrasonography should be scheduled before the GI procedure. Explanation: Both an upper GI procedure with barium ingestion and an ultrasonography may be completed on the same day. The ultrasonography test should be completed first, because the barium solution could interfere with the transmission of the sound waves. The ultrasonography test uses sound waves that are passed into internal body structures, and the echoes are recorded as they strike tissues. Fluid in the abdomen prevents transmission of ultrasound.

A nurse practitioner treating a patient who is diagnosed with hepatitis A should provide health care information. Which of the following statements are correct for this disorder? Select all that apply. The incubation period for this virus is up to 4 months. There is a 70% chance that jaundice will occur. Transmission of the virus is possible with oral-anal contact during sex. Typically there is a spontaneous recovery. There is a 50% risk that cirrhosis will develop.

There is a 70% chance that jaundice will occur. Transmission of the virus is possible with oral-anal contact during sex. Typically there is a spontaneous recovery. Explanation: The incubation period for hepatitis A is 15 to 50 days, with an average of 28 days. The risk of cirrhosis occurs with hepatitis B.

myocutaneous flap

This flap involves the transfer of intact muscle, subcutaneous tissue and skin as a single unit rotated on a relatively narrow blood supply of the muscle.

The nurse is caring for a patient with acute pancreatitis. The patient has an order for an anticholinergic medication. The nurse explains that the patient will be receiving that medication for what reason? To decrease metabolism To depress the central nervous system and increase the pain threshold To reduce gastric and pancreatic secretions To relieve nausea and vomiting

To reduce gastric and pancreatic secretions Explanation: Anticholinergic medications reduce gastric and pancreatic secretion.

Which of the following diagnostic studies definitely confirms the presence of ascites? Ultrasound of liver and abdomen Abdominal x-ray Colonoscopy Computed tomography of abdomen

Ultrasound of liver and abdomen Explanation: Ultrasonography of the liver and abdomen will definitively confirm the presence of ascites. An abdominal x-ray, colonoscopy, and computed tomography of the abdomen would not confirm the presence of ascites.

Which of the following is the most effective strategy to prevent hepatitis B infection? Vaccine Barrier protection during intercourse Covering open sores Avoid sharing toothbrushes

Vaccine Explanation: The most effective strategy to prevent hepatitis B infection is through vaccination. Recommendations to prevent transmission of hepatitis B include vaccination of sexual contacts of individuals with chronic hepatitis, use of barrier protection during sexual intercourse, avoidance of sharing toothbrushes, razors with others, and covering open sores or skin lesions.

Which medication is used to decrease portal pressure, halting bleeding of esophageal varices? Spironolactone Vasopressin Nitroglycerin Cimetidine

Vasopressin Explanation: Vasopressin may be the initial therapy for esophageal varices because it constricts the splanchnic arterial bed and decreases portal hypertension. Nitroglycerin has been used to prevent the side effects of vasopressin. Spironolactone and cimetidine do not decrease portal hypertension.

A client with severe and chronic liver disease is showing manifestations related to inadequate vitamin intake and metabolism. He reports difficulty driving at night because he cannot see well. Which of the following vitamins is most likely deficient for this client? Vitamin A Thiamine Riboflavin Vitamin K

Vitamin A Explanation: Problems common to clients with severe chronic liver dysfunction result from inadequate intake of sufficient vitamins. Vitamin A deficiency results in night blindness and eye and skin changes. Thiamine deficiency can lead to beriberi, polyneuritis, and Wernicke-Korsakoff psychosis. Riboflavin deficiency results in characteristic skin and mucous membrane lesions. Vitamin K deficiency can cause hypoprothrombinemia, characterized by spontaneous bleeding and ecchymoses.

A client tells the nurse that the stool was colored yellow. The nurse assesses the client for recent foods ingested. occult blood. ingestion of bismuth. pilonidal cyst.

recent foods ingested. Explanation: The nurse should assess for recent foods that the client ingested, as ingestion of senna can cause the stool to turn yellow. Ingestion of bismuth can turn the stool black and, when occult blood is present, the stool can appear to be tarry black.

A nurse is teaching a client with malabsorption syndrome about the disorder and its treatment. The client asks which part of the GI tract absorbs food. What is the nurse's best response? stomach small intestine large intestine rectum

small intestine Explanation: The small intestine absorbs products of digestion, completes food digestion, and secretes hormones that help control the secretion of bile, pancreatic juice, and intestinal secretions. The stomach stores, mixes, and liquefies the food bolus into chyme and controls food passage into the duodenum; it doesn't absorb products of digestion. Although the large intestine completes the absorption of water, chloride, and sodium, it plays no part in absorbing food. The rectum is the portion of the large intestine that forms and expels feces from the body; its functions don't include absorption.

A client with acute liver failure exhibits confusion, a declining level of consciousness, and slowed respirations. The nurse finds him very difficult to arouse. The diagnostic information which best explains the client's behavior is: elevated liver enzymes and low serum protein level. subnormal serum glucose and elevated serum ammonia levels. subnormal clotting factors and platelet count. elevated blood urea nitrogen and creatinine levels and hyperglycemia.

subnormal serum glucose and elevated serum ammonia levels. Explanation: In acute liver failure, serum ammonia levels increase because the liver can't adequately detoxify the ammonia produced in the GI tract. In addition, serum glucose levels decline because the liver isn't capable of releasing stored glucose. Elevated serum ammonia and subnormal serum glucose levels depress the level of a client's consciousness. Elevated liver enzymes, low serum protein level, subnormal clotting factors and platelet count, elevated blood urea nitrogen and creatine levels, and hyperglycemia aren't as directly related to the client's level of consciousness.

The nurse recognizes which change of the gastrointestinal system is an age-related change? increased motility hypertrophy of the small intestine weakened gag reflex increased mucus secretion

weakened gag reflex Explanation: A weakened gag reflex is an age-related change of the GI system. There is decreased motility, atrophy of the small intestine, and decreased mucus secretion.

A client with cholelithiasis has a gallstone lodged in the common bile duct. When assessing this client, the nurse expects to note: yellow sclerae. light amber urine. circumoral pallor. black, tarry stools.

yellow sclerae. Explanation: Yellow sclerae are an early sign of jaundice, which occurs when the common bile duct is obstructed. Urine normally is light amber. Circumoral pallor and black, tarry stools don't occur in common bile duct obstruction; they are signs of hypoxia and GI bleeding, respectively.

A client is prescribed tetracycline to treat peptic ulcer disease. Which instruction would the nurse give the client? A. "Take the medication with milk." B. "Be sure to wear sunscreen while taking this medicine." C. "Expect a metallic taste when taking this medicine, which is normal." D. "Do not drive when taking this medication."

"Be sure to wear sunscreen while taking this medicine." RATIONALE Tetracycline may cause a photosensitivity reaction in clients. The nurse should caution the client to use sunscreen when taking this drug. Dairy products can reduce the effectiveness of tetracycline, so the nurse should not advise him or her to take the medication with milk. A metallic taste accompanies administration of metronidazole (Flagyl). Administration of tetracycline does not necessitate driving restrictions.

The nurse is caring for a man who has experienced a spinal cord injury. Throughout his recovery, the client expects to gain control of his bowels. The nurse's best response to this client would be which of the following? "It is not going to happen. Your nerve cells are too damaged." "Having a bowel movement is a spinal reflex requiring intact nerve fibers. Yours are not intact." "Over time, the nerve fibers will regrow new tracts, and you can have bowel movements again." "Wearing an undergarment will become more comfortable over time."

"Having a bowel movement is a spinal reflex requiring intact nerve fibers. Yours are not intact." Explanation: The act of defecation is a spinal reflex involving the parasympathetic nerve fibers. Normally, the external anal sphincter is maintained in a state of tonic contraction. With a spinal cord injury, the client no longer has this nervous system control and is often incontinent.

A client discharged after a laparoscopic cholecystectomy calls the surgeon's office reporting severe right shoulder pain 24 hours after surgery. Which statement is the correct information for the nurse to provide to this client? "This pain is caused from the gas used to inflate your abdominal area during surgery. Sitting upright in a chair, walking, or using a heating pad may ease the discomfort." "This pain is caused from your incision. Take analgesics as needed and as prescribed and report to the surgeon if pain is unrelieved even with analgesic use." "This may be the initial symptoms of an infection. You need to come to see the surgeon today for an evaluation." "This pain may be caused by a bile duct injury. You will need to go to the hospital immediately to have this evaluated."

"This pain is caused from the gas used to inflate your abdominal area during surgery. Sitting upright in a chair, walking, or using a heating pad may ease the discomfort." Explanation: If pain occurs in the right shoulder or scapular area (from migration of the carbon dioxide used to insufflate the abdominal cavity during the procedure), the nurse may recommend using a heating pad for 15 to 20 minutes hourly, sitting up in a bed or chair, or walking.

Serologic testing of a middle-aged woman with a recent history of severe flu-like symptoms has just resulted in a diagnosis of hepatitis A. Which of the following assessment questions should the nurse prioritize when discussing this diagnosis with the patient? "Are you currently in a monogamous sexual relationship?" "How would describe your typical diet? "Which restaurants have you eaten in over the past few weeks?" "Have you ever used recreational drugs?"

"Which restaurants have you eaten in over the past few weeks?" Explanation: The mode of transmission of hepatitis A virus (HAV) occurs through the fecal-oral route, primarily through person-to-person contact and/or ingestion of fecally contaminated food or water. Uncooked food or poor food handling practice is a common method of transmission of HAV. As a result, the patient is usually questioned about his or her recent restaurant visits. HAV is less commonly related to sexual contact, drug use, or poor diet.

A client has a cheesy white plaque in the mouth. The plaque looks like milk curds and can be rubbed off. What is the nurse's best intervention? 1- Instruct the client to swish prescribed nystatin solution for 1 minute. 2- Remove the plaque from the mouth by rubbing with gauze. 3- Provide saline rinses prior to meals. 4- Encourage the client to ingest a soft or bland diet.

1

A client has a new order for metoclorpramide. What potential side effects should the nurse educate the client about? 1- Extrapyramidal 2- Peptic ulcer disease 3- Gastric slowing 4- Nausea

1

A client is receiving insulin lispro at 7:30 AM. The nurse ensures that the client has breakfast by which time? 1- 7:45 AM 2- 8:00 AM 3- 8:15 AM 4- 8:30 AM

1

A client is receiving parenteral nutrition (PN) through a peripherally inserted central catheter (PICC) and will be discharged home with PN. The home health nurse evaluates the home setting and would make a recommendation when noticing which circumstance? 1- No land line; cell phone available and taken by family member during working hours 2- Water of low pressure that can be obtained through all faucets 3- Little food in the working refrigerator 4- Electricity that loses power, usually for short duration, during storms

1

A client who can't tolerate oral feedings begins receiving intermittent enteral feedings. When monitoring for evidence of intolerance to these feedings, what must the nurse remain alert for? 1- diaphoresis, vomiting, and diarrhea. 2- manifestations of electrolyte disturbances. 3- manifestations of hypoglycemia. 4- constipation, dehydration, and hypercapnia.

1

A nurse is caring for a diabetic patient with a diagnosis of nephropathy. What would the nurse expect the urinalysis report to indicate? 1- Albumin 2- Bacteria 3- Red blood cells 4- White blood cells

1

A nurse is conducting morning assessments of several medical patients and has entered the room of a patient who has a nasogastric (NG) tube in situ. Immediately, the nurse observes that the tube has become unsecured from the patient's nose and the mark at the desired point of entry is now approximately 8 inches from the patient's nose. How should the nurse best respond to this assessment finding? 1- Reinsert the NG tube and arrange for x-ray confirmation of placement. 2- Remove the NG tube and obtain an order for reinsertion. 3- Reinsert the NG tube and monitor the patient closely for signs of aspiration. 4- Reinsert the NG tube and aspirate stomach contents to confirm correct placement.

1

A nurse is preparing a continuous insulin infusion for a child with diabetic ketoacidosis and a blood glucose level of 800 mg/dl. Which solution is the most appropriate at the beginning of therapy? 1- 100 units of regular insulin in normal saline solution 2- 100 units of neutral protamine Hagedorn (NPH) insulin in normal saline solution 3- 100 units of regular insulin in dextrose 5% in water 4- 100 units of NPH insulin in dextrose 5% in water

1

A nurse is teaching a client about insulin infusion pump use. What intervention should the nurse include to prevent infection at the injection site? 1- Change the needle every 3 days. 2- Wear sterile gloves when inserting the needle. 3- Take the ordered antibiotics before initiating treatment. 4- Use clean technique when changing the needle.

1

A patient has been NPO for two days anticipating surgery which has been repeatedly delayed. In addition to risks of nutritional and fluid deficits, the nurse determines that this patient is at the greatest risk for: 1- altered oral mucous membranes. 2- physical injury. 3- ineffective social interaction. 4- confusion.

1

A public health nurse is participating in a community health fair that is focused on health promotion and illness prevention. Which of the following older adults most likely faces the highest risk of developing oral cancer? 1- A man who describes himself as always having been a "heavy smoker and a heavy drinker." 2- A woman who is morbidly obese and has a longstanding diagnosis of systemic lupus erythematosus (SLE). 3- A woman who describes herself as a "proud breast cancer survivor for over 10 years." 4- A man who states that he enjoys good health, with the exception of "heartburn after nearly every meal."

1

An elderly patient comes into the emergency department complaining of an earache. The patient has an oral temperature of 100.2° F. Otoscopic assessment of the ear reveals a pearly gray tympanic membrane with no evidence of discharge or inflammation. Which action should the triage nurse take next? 1- Palpate the patient's parotid glands to detect swelling and tenderness. 2- Assess the temporomandibular joint for evidence of a malocclusion. 3- Test the integrity of the 12th cranial nerve by asking the patient to protrude his tongue. 4- Inspect the patient's gums for bleeding and hyperpigmentation.

1

Gastrostomy feedings are preferred to nasogastric feedings in the comatose patient, because the: 1- Gastroesophageal sphincter is intact, lessening the possibility of regurgitation and aspiration. 2- Digestive process occurs more rapidly as a result of the feedings not having to pass through the esophagus. 3- Feedings can be administered with the patient in the recumbent position. 4- The patient cannot experience the deprivational stress of not swallowing.

1

The greatest percentage of people have which type of diabetes? 1- Type 2 2- Type 1 3- Gestational 4- Impaired glucose tolerance

1

The nurse cares for a client who receivies continuous enteral tube feedings and who is at low risk for aspiration. The nurse assesses the gastric residual volume to be 350 mL. The nurse determines which action is correct? 1- Monitoring the feeding closely. 2- Increasing the feeding rate. 3- Lowering the head of the bed. 4- Flushing the feeding tube.

1

The nurse caring for a patient who is being discharged home after a radical neck dissection has worked with the home health nurse to develop a plan of care for this patient. What is a priority psychosocial outcome for a patient who has had a radical neck dissection? 1- Indicates acceptance of altered appearance and demonstrates positive self-image 2- Freely expresses needs and concerns related to postoperative pain management 3- Compensates effectively for alteration in ability to communicate related to dysarthria 4- Demonstrates effective stress management techniques to promote muscle relaxation

1

The nurse conducts discharge education for a client who is to go home with parenteral nutrition (PN). The nurse determines the client understands the education when the client indicates a sign and/or symptom of metabolic complications is 1- loose, watery stools. 2- increased urination. 3- elevated blood pressure. 4- decreased pulse rate.

1

The primary source of microorganisms for catheter-related infections are the skin and which of the following? 1- Catheter hub 2- Catheter tubing 3- IV fluid bag 4- IV tubing

1

What is the duration of regular insulin? 1- 4 to 6 hours 2- 3 to 5 hours 3- 12 to 16 hours 4- 24 hours

1

When assessing a client during a routine checkup, the nurse reviews the history and notes that the client had aphthous stomatitis at the time of the last visit. How is aphthous stomatitis best described by the nurse? 1- A canker sore of the oral soft tissues 2- An acute stomach infection 3- Acid indigestion 4- An early sign of peptic ulcer disease

1

Which is the primary symptom of achalasia? 1- Difficulty swallowing 2- Chest pain 3- Heartburn 4- Pulmonary symptoms

1

Which of the following insulins are used for basal dosage? 1- Glarginet (Lantus) 2- NPH (Humulin N) 3- Lispro (Humalog) 4- Aspart (Novolog)

1

Which of the following is one of the first clinical manifestations of esophageal cancer? 1- Increasing difficulty in swallowing 2- Sensation of a mass in throat 3- Foul breath 4- Hiccups

1

A nurse is caring for a client receiving parenteral nutrition at home. The client was discharged from the acute care facility 4 days ago. What would the nurse include in the client's plan of care? Select all that apply. 1- Daily weights 2- Intake and output monitoring 3- Calorie counts for oral nutrients 4- Daily transparent dressing changes 5- Strict bedrest

1,2,3

A nurse is assigned to care for a patient who is suspected of having type 2 diabetes. Select all the clinical manifestations that the nurse knows could be consistent with this diagnosis. 1- Blurred or deteriorating vision 2- Fatigue and irritability 3- Polyuria and polydipsia 4- Sudden weight loss and anorexia 5- Wounds that heal slowly or respond poorly to treatment

1,2,3,5

A patient is diagnosed with type 1 diabetes. What clinical characteristics does the nurse expect to see in this patient? Select all that apply. 1- Ketosis-prone 2- Little endogenous insulin 3- Obesity at diagnoses 4- Younger than 30 years of age 5- Older than 65 years of age

1,2,4

A client experienced surgical resection of a tumor of the esophagus. After recovery from the anesthesia, what will the nurse include in the postoperative care plans? Select all that apply. 1- Assess lung sounds every 4 hours and prn. 2- Replace the nasogastric tube if the tube becomes dislodged. 3- Monitor drainage in the closed chest drainage system. 4- Verify rhythm on the cardiac monitoring system. 5- Maintain the client in a side-lying position.

1,3,4

A 54 year-old man is postoperative day 1 following neck dissection surgery. Which of the following nursing actions should the nurse prioritize in the care of this patient? 1- Teaching the patient about the signs and symptoms of major postoperative complications 2- Positioning the patient in a high Fowler's position to protect the airway 3- Ensuring that naloxone (Narcan) is available at the patient's bedside 4- Maintaining protective isolation for 24 to 36 hours after surgery

2

A 59-year-old woman with a recent history of heartburn, regurgitation, and occasional dysphagia has been diagnosed with a sliding hiatal hernia following an upper GI series. The nurse is providing patient education about the management of this health problem. What should the nurse suggest as a management strategy to this patient? 1- Minimizing her intake of highly spiced foods and dairy products 2- Remaining upright for at least 1 hour following each meal 3- Abstaining from alcohol 4- Drinking one to two glasses of water before and after each meal

2

A child is brought into the emergency department with vomiting, drowsiness, and blowing respirations. The father reports that the symptoms have been progressing throughout the day. The nurse suspects diabetic ketoacidosis (DKA). Which action should the nurse take first in the management of DKA? 1- Give prescribed antiemetics. 2- Begin fluid replacements. 3- Administer prescribed dose of insulin. 4- Administer bicarbonate to correct acidosis.

2

A client recovering from gastric bypass surgery accidentally removes the nasogastric tube. What is the best action by the nurse? 1- Reinsert the nasogastric tube to the stomach. 2- Notify the surgeon about the tube's removal. 3- Place the nasogastric tube to the level of the esophagus. 4- Document the discontinuation of the nasogastric tube.

2

A client who was diagnosed with type 1 diabetes 14 years ago is admitted to the medical-surgical unit with abdominal pain. On admission, the client's blood glucose level is 470 mg/dl. Which finding is most likely to accompany this blood glucose level? 1- Cool, moist skin 2- Rapid, thready pulse 3- Arm and leg trembling 4- Slow, shallow respirations

2

A nurse caring for a client who has had radical neck surgery notices an abnormal amount of serosanguineous secretions in the wound suction unit during the first postoperative day. What is an expected, normal amount of drainage? 1- Between 40 and 80 mL 2- Approximately 80 to 120 mL 3- Between 120 and 160 mL 4- Greater than 160 mL

2

A patient has just been diagnosed with type 2 diabetes. The health care provider has prescribed an oral antidiabetic agent that will inhibit the production of glucose by the liver and thereby aid in the control of blood glucose. What type of oral antidiabetic agent has been prescribed for this patient? 1- Sulfonylurea 2- Biguanide 3- Thiazolidinedione 4- Alpha glucosidase inhibitor

2

A patient is receiving parenteral nutrition. The current solution is nearing completion, and a new solution is to be hung, but it has not arrived from the pharmacy. Which action by the nurse would be most appropriate? 1- Slow the current infusion rate so that it will last until the new solution arrives. 2- Hang a solution of dextrose 10% and water until the new solution is available. 3- Have someone go to the pharmacy to obtain the new solution. 4- Begin an infusion of normal saline in another site to maintain hydration.

2

A patient with a recent diagnosis of esophageal cancer has undergone an esophagectomy and is currently receiving care in a step-down unit. The nurse in the step-down unit is aware of the specific complications associated with this surgical procedure and is consequently monitoring the patient closely for signs and symptoms of: 1- Increased intracranial pressure (ICP) 2- Aspiration pneumonia 3- Abdominal aortic aneurysm (AAA) 4- Dyspepsia

2

An agitated, confused client arrives in the emergency department. The client's history includes type 1 diabetes, hypertension, and angina pectoris. Assessment reveals pallor, diaphoresis, headache, and intense hunger. A stat blood glucose sample measures 42 mg/dl, and the client is treated for an acute hypoglycemic reaction. After recovery, the nurse teaches the client to treat hypoglycemia by ingesting: 1- 2 to 5 g of a simple carbohydrate. 2- 10 to 15 g of a simple carbohydrate. 3- 18 to 20 g of a simple carbohydrate. 4- 25 to 30 g of a simple carbohydrate.

2

An older adult patient is in the hospital being treated for sepsis related to a urinary tract infection. The patient has started to have an altered sense of awareness, profound dehydration, and hypotension. What does the nurse suspect the patient is experiencing? 1- Systemic inflammatory response syndrome 2- Hyperglycemic hyperosmolar syndrome 3- Multiple-organ dysfunction syndrome 4- Diabetic ketoacidosis

2

Postoperatively, a client with a radical neck dissection should be placed in which position? 1- Supine 2- Fowler 3- Prone 4- Side-lying

2

The nurse confirms placement of a client's nasogastric (NG) tube using a combination of visual and pH assessment of the aspirate. The nurse determines that the NG tube remains properly placed when the pH of the aspirate is 1- alkaline 2- acidic 3- neutral 4- unmeasurable

2

The nurse is creating a plan of care for a client who is not able to tolerate brushing his teeth. The nurse includes which mouth irrigation in the plan of care? 1- Dextrose and water 2- Baking soda and water 3- Full-strength peroxide 4- Mouthwash and water

2

The nurse is preparing a presentation for a group of adults at a local community center about diabetes. Which of the following would the nurse include as associated with type 2 diabetes? 1- Onset most common during adolescence 2- Insulin production insufficient 3- Less common than type 1 diabetes 4- Little to relation to pre-diabetes

2

The nurse observes dry mucous membranes in a client who is receiving tube feedings after an oral surgery. The client also reports unpleasant tastes and odors. Which measure should be included in the client's plan of care? 1- Ensure adequate hydration with additional water. 2- Provide frequent mouth care. 3- Keep the feeding formula refrigerated. 4- Flush the tube with water before adding the feedings.

2

To prevent gastroesophageal reflux in a client with hiatal hernia, the nurse should provide which discharge instruction? 1- "Lie down after meals to promote digestion." 2- "Avoid coffee and alcoholic beverages." 3- "Take antacids with meals." 4- "Limit fluid intake with meals."

2

Which condition is caused by improper catheter placement and inadvertent puncture of the pleura? 1- air embolism 2- pneumothorax 3- sepsis 4- fluid overload

2

A client is admitted to the health care center with abdominal pain, nausea, and vomiting. The medical reports indicate a history of type 1 diabetes. The nurse suspects the client's symptoms to be those of diabetic ketoacidosis (DKA). Which action will help the nurse confirm the diagnosis? 1- Assess the client's ability to take a deep breath 2- Assess the client's ability to move all extremities 3- Assess the client's breath odor 4- Assess for excessive sweating

3

A client who reports increasing difficulty swallowing, weight loss, and fatigue is diagnosed with esophageal cancer. Because this client has difficulty swallowing, what should the nurse assign highest priority to? 1- Helping the client cope with body image changes 2- Ensuring adequate nutrition 3- Maintaining a patent airway 4- Preventing injury

3

A client with a gastrojejunostomy is beginning to take solid food. Which finding would lead the nurse to suspect that the client is experiencing dumping syndrome? 1- Dry skin 2- Slowed heart beat 3- Diarrhea 4- Hyperglycemia

3

A client with long-standing type 1 diabetes is admitted to the hospital with unstable angina pectoris. After the client's condition stabilizes, the nurse evaluates the diabetes management regimen. The nurse learns that the client sees the physician every 4 weeks, injects insulin after breakfast and dinner, and measures blood glucose before breakfast and at bedtime. Consequently, the nurse should formulate a nursing diagnosis of: 1- Impaired adjustment. 2- Defensive coping. 3- Deficient knowledge (treatment regimen). 4- Health-seeking behaviors (diabetes control).

3

A nurse obtains a fingerstick glucose level of 45 mg/dl on a client newly diagnosed with diabetes mellitus. The client is alert and oriented, and the client's skin is warm and dry. How should the nurse intervene? 1- Give the client 4 oz of milk and a graham cracker with peanut butter. 2- Obtain a serum glucose level. 3- Obtain a repeat fingerstick glucose level. 4- Notify the physician.

3

A nurse practitioner, who is treating a patient with GERD, knows that responsiveness to this drug classification is validation of the disease. The drug classification is: 1- H2-receptor antagonists. 2- Antispasmodics 3- Proton pump inhibitors. 4- Antacids

3

A patient has been diagnosed with a hiatal hernia. The nurse explains the diagnosis to the patient and his family by telling them that a hernia is a (an): 1- Extension of the esophagus through an opening in the diaphragm. 2- Involution of the esophagus, which causes a severe stricture. 3- Protrusion of the upper stomach into the lower portion of the thorax. 4- Twisting of the duodenum through an opening in the diaphragm.

3

An older adult patient who has been living at home alone is diagnosed with parotitis. What causative bacteria does the nurse suspect is the cause of the parotitis? 1- Methicillin-resistant Streptococcus aureus (MRSA) 2- Pneumococcus 3- Staphylococcus aureus 4- Streptococcus viridans

3

Insulin is a hormone secreted by the Islets of Langerhans and is essential for the metabolism of carbohydrates, fats, and protein. The nurse understands the physiologic importance of gluconeogenesis, which refers to the: 1- Transport of potassium. 2- Release of glucose. 3- Synthesis of glucose from noncarbohydrate sources. 4- Storage of glucose as glycogen in the liver.

3

Laboratory studies indicate a client's blood glucose level is 185 mg/dl. Two hours have passed since the client ate breakfast. Which test would yield the most conclusive diagnostic information about the client's glucose use? 1- Fasting blood glucose test 2- 6-hour glucose tolerance test 3- Serum glycosylated hemoglobin (Hb A1c) 4- Urine ketones

3

Select the assessment finding that the nurse should immediately report, post radical neck dissection. 1- Temperature of 99°F 2- Pain 3- Stridor 4- Localized wound tenderness

3

The client is experiencing swallowing difficulties and is now scheduled to receive a gastric feeding. The client has the following oral medications prescribed: furosemide, digoxin, enteric coated aspirin, and vitamin E. The nurse would withhold which medication? 1- furosemide 2- digoxin 3- enteric coated aspirin 4- vitamin E

3

The most common symptom of esophageal disease is 1- nausea. 2- vomiting. 3- dysphagia. 4- odynophagia.

3

The nurse is checking placement of a nasogastric (NG) tube that has been in place for 2 days. The tube is draining green aspirate. What does this color of aspirate indicate? 1- The tube is in the pleural space. 2- The tube is the intestine. 3- The tube is in the stomach. 4- The tube is in the esophagus.

3

The nurse is inserting a Levin tube for a patient for gastric decompression. The tube should be inserted to 6 to 10 cm beyond what length? 1- A length of 50 cm (20 in) 2- A point that equals the distance from the nose to the xiphoid process 3- The distance measured from the tip of the nose to the earlobe and from the earlobe to the xiphoid process 4- The distance determined by measuring from the tragus of the ear to the xiphoid process

3

The nurse is preparing to assess the donor site of a client who underwent a myocutaneous flap after a radical neck dissection. The nurse prepares to assess the most commonly used muscle for this surgery. Which muscle should the nurse assess? 1- Trapezius 2- Biceps 3- Pectoralis major 4- Sternomastoid

3

The nurse prepares to administer all of a client's medications via feeding tube. The nurse consults the pharmacist and/or physician when the nurse notes which type of oral medication on the client's medication administration record? 1- simple compressed tablets 2- buccal or sublingual tablets 3- enteric-coated tablets 4- soft, gelatin capsules filled with liquid

3

The nurse working in the recovery room is caring for a client who had a radical neck dissection. The nurse notices that the client makes a coarse, high-pitched sound upon inspiration. Which intervention by the nurse is appropriate? 1- Document the presence of stridor 2- Administer a breathing treatment 3- Notify the physician 4- Lower the head of the bed

3

Total parenteral nutrition (TPN) has been ordered for a male patient who has been experiencing a severe and protracted exacerbation of Crohn's disease. Before TPN can be initiated, the patient requires: 1- A random blood glucose level of ≤160 mg/dL 2- Angiography to determine the patency of his vascular system 3- The insertion of a central venous access device 4- A fluid challenge to assess his renal function

3

Which instruction should a nurse give to a client with diabetes mellitus when teaching about "sick day rules"? 1- "Don't take your insulin or oral antidiabetic agent if you don't eat." 2- "It's okay for your blood glucose to go above 300 mg/dl while you're sick." 3- "Test your blood glucose every 4 hours." 4- "Follow your regular meal plan, even if you're nauseous."

3

Which is the best nursing explanation for the symptom of polyuria in a client with diabetes mellitus? 1- With diabetes, drinking more results in more urine production. 2- Increased ketones in the urine promote the manufacturing of more urine. 3- High sugar pulls fluid into the bloodstream, which results in more urine production. 4- The body's requirement for fuel drives the production of urine.

3

A nurse should monitor blood glucose levels for a patient diagnosed with hyperinsulinism. What blood value does the nurse recognize as inadequate to sustain normal brain function? 30 mg/dL 50 mg/dL 70 mg/dL 90 mg/dL

30 mg/dL Explanation: Hyperinsulinism is caused by overproduction of insulin by the pancreatic islets. Occasionally, tumors of nonpancreatic origin produce an insulin-like material that can cause severe hypoglycemia and may be responsible for seizures coinciding with blood glucose levels that are too low to sustain normal brain function (i.e., lower than 30 mg/dL [1.6 mmol/L]) (Goldman & Schafer, 2012; McPherson & Pincus, 2011).

A 30-year-old type 1 diabetic has been admitted to the critical care unit with a diagnosis of diabetic ketoacidosis following a drinking binge over the course of a weekend. The nurse should anticipate that this patient will require what immediate intervention? 1- IV administration of calcium gluconate 2- Subcutaneous administration of 30 units of insulin glargine (Lantus) 3- Oral administration of 2 g of metformin (Glucophage) 4- Rapid administration of intravenous normal saline

4

A client has a radical neck dissection to treat cancer of the neck. The nurse develops the care plan and includes all the following diagnoses. The nurse identifies the highest priority diagnosis as 1- Impaired tissue integrity related to surgical intervention 2- Imbalanced nutrition: less than body requirements, related to treatment 3- Risk for infection related to surgical intervention 4- Ineffective airway clearance related to obstruction by mucus

4

A client with a tentative diagnosis of hyperosmolar hyperglycemic nonketotic syndrome (HHNS) has a history of type 2 diabetes that is being controlled with an oral diabetic agent, tolazamide. Which laboratory test is the most important for confirming this disorder? 1- Serum potassium level 2- Serum sodium level 3- Arterial blood gas (ABG) values 4- Serum osmolarity

4

A nurse enters the room of a client who has returned to the unit after having a radical neck dissection. Which assessment finding requires immediate intervention? 1- Serosanguineous drainage on the dressing 2- Foley catheter bag containing 500 ml of amber urine 3- A piggyback infusion of levofloxacin 4- The client lying in a lateral position, with the head of bed flat

4

A nurse expects to find which signs and symptoms in a client experiencing hypoglycemia? 1- Polyuria, headache, and fatigue 2- Polyphagia and flushed, dry skin 3- Polydipsia, pallor, and irritability 4- Nervousness, diaphoresis, and confusion

4

A nurse is assigned to care for a postoperative client with diabetes mellitus. During the assessment interview, the client reports that he's impotent and says he's concerned about the effect on his marriage. In planning this client's care, the most appropriate intervention would be to: 1- encourage the client to ask questions about personal sexuality. 2- provide time for privacy. 3- provide support for the spouse or significant other. 4- suggest referral to a sex counselor or other appropriate professional.

4

A nurse knows to assess a patient with type 1 diabetes for postprandial hyperglycemia. The nurse knows that glycosuria is present when the serum glucose level exceeds: 1- 120 mg/dL 2- 140 mg/dL 3- 160 mg/dL 4- 180 mg/dL

4

A patient who is HIV positive comes to the clinic and is experiencing white patches with rough hairlike projections. The nurse observes the lesions on the lateral border of the tongue. What abnormality of the mouth does the nurse determine these lesions are? 1- Aphthous stomatitis 2- Nicotine stomatitis 3- Erythroplakia 4- Hairy leukoplakia

4

An elderly client seeks medical attention for a vague complaint of difficulty swallowing. Which of the following assessment findings is most significant as related to this symptom? 1- Hiatal hernia 2- Gastroesophageal reflux disease 3- Gastritis 4- Esophageal tumor

4

The nurse is educating a patient about the benefits of fruit versus fruit juice in the diabetic diet. The patient states, "What difference does it make if you drink the juice or eat the fruit? It is all the same." What is the best response by the nurse? 1- "Eating the fruit is more satisfying than drinking the juice. You will get full faster." 2- "Eating the fruit will give you more vitamins and minerals than the juice will." 3- "The fruit has less sugar than the juice." 4- "Eating the fruit instead of drinking juice decreases the glycemic index by slowing absorption."

4

Which category of oral antidiabetic agents exerts the primary action by directly stimulating the pancreas to secrete insulin? 1- Thiazolidinediones 2- Biguanides 3- Alpha-glucosidase inhibitors 4- Sulfonylureas

4

Which is an accurate statement regarding cancer of the esophagus? 1- It is three times more common in women than men in the United States. 2- It is seen more frequently in Caucasian Americans than in African Americans. 3- It usually occurs in the fourth decade of life. 4- Chronic irritation of the esophagus is a known risk factor.

4

administered over a period of less than 20 minutes. What is a nursing measure to prevent or minimize the dumping syndrome? 1- Administer the feeding at a warm temperature to decrease peristalsis. 2- Administer the feeding by bolus to prevent continuous intestinal distention. 3- Administer the feeding with about 100 mL of fluid to dilute the high carbohydrate concentration. 4- Administer the feeding with the patient in semi-Fowler's position to decrease transit time influenced by gravity.

4

The nurse knows that the serum amylase concentration returns to normal within which time frame? 12 hours 24 hours 36 hours 48 hours

48 hours Explanation: Serum amylase usually returns to normal within 48 to 72 hours.

A client with Addison's disease comes to the clinic for a follow-up visit. When assessing this client, the nurse should stay alert for signs and symptoms of: A. sodium and potassium abnormalities. B sodium and chloride abnormalities. C. calcium and phosphorus abnormalities. D chloride and magnesium abnormalities.

A

A client has been prescribed a protein intake of 0.6 g/kg of body weight. The client weighs 154 pounds. The nurse calculates the daily protein intake to be how many grams? Enter the correct number ONLY.

42

Pharmacologic therapy frequently is used to dissolve small gallstones. It takes about how many months of medication with UDCA or CDCA for stones to dissolve? 1 to 2 3 to 5 6 to 12 13 to 18

6 to 12 Explanation: Ursodeoxycholic acid (UDCA [URSO, Actigall]) and chenodeoxycholic acid (chenodiol or CDCA [Chenix]) have been used to dissolve small, radiolucent gallstones composed primarily of cholesterol. Six to 12 months of therapy are required in many clients to dissolve stones, and monitoring of the client for recurrence of symptoms or occurrence of side effects (e.g., GI symptoms, pruritus, headache) is required during this time.

A nurse in a large university hospital has cared for several patients with endocrine disorders over the past year. For which of the following patients would a nursing diagnosis of disturbed body image be most likely applicable? A. A woman with a longstanding diagnosis of Cushing's syndrome B A man who was treated for Hashimoto's thyroiditis C A man who was diagnosed with hypoparathyroidism after neck surgery D A woman whose diagnosis of Graves' disease required radioactive iodine therapy

A

A nurse is preparing an IV dose of hydrocortisone that is to be administered to an adult patient on an acute medical unit. The endocrine disorder for which this treatment is most clearly indicated is: A Addison's disease B. Cushing's syndrome C. Diabetes insipidus D. Syndrome of inappropriate antidiuretic hormone (SIADH)

A

The nurse is instructing the client on frequent sensations experienced when a contrast agent is injected into the body during diagnostic studies. Which sensation is most common? Light-headedness A warm sensation Heart palpitations Chills

A warm sensation Explanation: The nurse informs the client that he or she may experience a warm sensation and nausea when the contrast agent is instilled. The client is instructed to take a couple of deep breaths, and, many times, the sensation will go away. The other options are not frequently encountered.

Endoscopy of a 60-year-old woman has revealed the presence of an esophageal peptic ulcer. The nurse who is providing this woman's care is assessing for risk factors that may have contributed to the development of this disease. What question most directly addresses these risk factors? A. "Have you ever been diagnosed with reflux?" B. "Do you consider yourself to have a healthy diet?" C. "Have you been prone to infections over the past few years?" D. "Do you ever find it difficult to swallow certain foods?"

A. "Have you ever been diagnosed with reflux?" RATIONALE Gastroesophageal reflux disease (GERD) is a significant risk factor for peptic ulcer disease. Poor diet, general infections, and dysphagia are less closely associated with etiology of esophageal ulcers. Reference:

A patient has been diagnosed with acute gastritis and asks the nurse what could have caused it. What is the best response by the nurse? (Select all that apply.) A. "It can be caused by ingestion of strong acids." B. "You may have ingested some irritating foods." C. "Is it possible that you are overusing aspirin." D. "It is a hereditary disease." E. "It is probably your nerves."

A. "It can be caused by ingestion of strong acids." B. "You may have ingested some irritating foods." C. "Is it possible that you are overusing aspirin." RATIONALE Acute gastritis is often caused by dietary indiscretion—the person eats food that is irritating, too highly seasoned, or contaminated with disease-causing microorganisms. Other causes of acute gastritis include overuse of aspirin and other nonsteroidal anti-inflammatory drugs (NSAIDs), excessive alcohol intake, bile reflux, and radiation therapy. A more severe form of acute gastritis is caused by the ingestion of strong acid or alkali, which may cause the mucosa to become gangrenous or to perforate.

Which of the following appears to be a significant factor in the development of gastric cancer? A. Diet B. Age C. Ethnicity D. Gender

A. Diet RATIONALE Diet seems to be a significant factor: a diet high in smoked, salted, or pickled foods and low in fruits and vegetables may increase the risk of gastric cancer. The typical patient with gastric cancer is between 50 and 70 years of age. Men have a higher incidence than women. Native Americans, Hispanic Americans, and African Americans are twice as likely as Caucasian Americans to develop gastric cancer.

Which medication should the nurse question before administering it to a patient with peptic ulcer disease? A. E-mycin, an antibiotic B. Prilosec, a PPI C. Flagyl, an antimicrobial agent D. Tylenol, a nonnarcotic analgesic

A. E-mycin, an antibiotic RATIONALE The antibiotic E-mycin is irritating to the GI tract and should be questioned in a patient with PUD.

The nurse is assessing a client with an ulcer for signs and symptoms of hemorrhage. The nurse interprets which condition as a sign/symptom of possible hemorrhage? A. Hematemesis B. Bradycardia C. Hypertension D. Polyuria

A. Hematemesis RATIONALE The nurse interprets hematemesis as a sign/symptom of possible hemorrhage from the ulcer. Other signs that can indicate hemorrhage include tachycardia, hypotension, and oliguria/anuria.

A patient presents to the walk-in clinic complaining of vomiting and burning in his mid-epigastria. The nurse knows that to confirm peptic ulcer disease, the health care provider is likely to order a diagnostic test to detect the presence of what? A. Infection with Helicobacter pylori B. Excessive stomach acid secretion C. Gastric irritation caused by nonsteroidal anti-inflammatory drugs (NSAIDs) D. Inadequate production of pancreatic enzymes

A. Infection with Helicobacter pylori RATIONALE H. pylori infection may be determined by endoscopy and histologic examination of a tissue specimen obtained by biopsy, or a rapid urease test of the biopsy specimen. Other less invasive diagnostic measures for detecting H. pylori include serologic testing for antibodies against the H. pylori antigen, stool antigen test, and urea breath test. Excessive stomach acid secretion, NSAIDs, and dietary indiscretion may all cause gastritis; however, peptic ulcers are caused by colonization of the stomach by H. pylori. Reference:

A 32-year-old man who has a body mass index of 32 (morbidly obese) is considering bariatric surgery. In the time leading up to this surgery, which of the following nursing diagnoses will be the primary focus of interventions? A. Knowledge deficit related to the implications of bariatric surgery B. Altered growth and development related to obesity C. Risk for injury related to obesity D. Spiritual distress related to low body image

A. Knowledge deficit related to the implications of bariatric surgery RATIONALE Patient teaching is a priority in the preparation for bariatric surgery. Necessary counseling and education would be prioritized over growth and development and spiritual distress, although each may emerge and be addressed accordingly. In seeking bariatric surgery, the patient is likely already aware of the risks associated with obesity.

A 75-year-old male patient presents at the emergency department with symptoms of a small bowel obstruction. An emergency room nurse is obtaining assessment data from this patient. What assessment finding is characteristic of a small bowel obstruction? A. Nausea and vomiting B. Decrease in urine production C. Mucus in the stool D. Mucosal edema

A. Nausea and vomiting RATIONALE Nausea and vomiting are symptoms of a small bowel obstruction. Decrease in urine production and mucosal edema are not symptoms of a bowel obstruction. The patient may defecate mucus, but this is not accompanied by stool.

Which medication classification represents a proton (gastric acid) pump inhibitor? A. Omeprazole B. Sucralfate C. Famotidine D. Metronidazole

A. Omeprazole RATIONALE Omeprazole decreases gastric acid by slowing the hydrogen-potassium adenosine triphosphatase pump on the surface of the parietal cells. Sucralfate is a cytoprotective drug. Famotidine is a histamine-2 receptor antagonist. Metronidazole is an antibiotic, specifically an amebicide.

Which of the following foods could give a false-positive result on the fecal occult blood test (FOBT)? Select all that apply. A. Red meats B. Pasta C. Turnips D. Fish E. Whole-grain bread

A. Red meats C. Turnips D. Fish RATIONALE Fish, red meats, and turnips can produce false-positive results if consumed prior to the collection of stool for fecal occult blood testing (FOBT). Pasta and whole grain bread do not affect the results of the FOBT.

Which of the following is the most successful treatment for gastric cancer? A. Removal of the tumor B. Chemotherapy C. Radiation D. Palliation

A. Removal of the tumor RATIONALE There is no successful treatment for gastric carcinoma except removal of the tumor. If the tumor can be removed while it is still localized to the stomach, the patient may be cured. If the tumor has spread beyond the area that can be excised, cure is less likely.

The nurse is providing care to a client who has had a percutaneous liver biopsy. The nurse would monitor the client for which of the following? A. Signs and symptoms of bleeding B. Return of the gag reflex C. Passage of stool D. Intake and output

A. Signs and symptoms of bleeding RATIONALE A major complication after a liver biopsy is bleeding, so it would be important for the nurse to monitor the client for signs and symptoms of bleeding. Return of the gag reflex would be important for the client who had an esophagogastroduodenoscopy to prevent aspiration. Monitoring the passage of stool would be important for a client who had a barium enema or colonoscopy. Monitoring intake and output is a general measure indicated for any client. It is not specific to a liver biopsy.

A nurse is providing care for a client recovering from gastric bypass surgery. During assessment, the client exhibits pallor, perspiration, palpitations, headache, and feelings of warmth, dizziness, and drowsiness. The client reports eating 90 minutes ago. What will the nurse suspect? A. Vasomotor symptoms associated with dumping syndrome B. Dehiscence of the surgical wound C. Peritonitis D. A normal reaction to surgery

A. Vasomotor symptoms associated with dumping syndrome RATIONALE Early manifestations of dumping syndrome occur 15 to 30 minutes after eating. Signs and symptoms include vertigo, tachycardia, syncope, sweating, pallor, palpitations, diarrhea, nausea, and the desire to lie down. Dehiscence of the surgical wound is characterized by pain and a pulling or popping feeling at the surgical site. Peritonitis presents with a rigid, boardlike abdomen, tenderness, and fever. The client's signs and symptoms aren't a normal reaction to surgery. Reference:

A client is scheduled to undergo rhinoplasty in the morning, and reports medications used on a daily basis, which the nurse records on the client's chart. Which daily medications have the potential to result in constipation? A. laxative B. multivitamin without iron C. NSAIDs D. acetaminophen

A. laxative RATIONALE Constipation may also result from chronic use of laxatives ("cathartic colon")because such use can cause a loss of normal colonic motility and intestinal tone. Laxatives also dull the gastrocolic reflex.

A nurse assesses the stools of a client diagnosed with peptic ulcer disease. Inspection reveals black, tarry stools. The nurse would use which term to document this finding? A. melana B. hematemesis C. pyrosis D. achlorhydria

A. melana RATIONALE Melena is the term used to denote black, tarry stools. Hematemesis refers to blood in vomit. Pyrosis is a burning sensation in the esophagus and stomach that moves up to the mouth. Achlorhydria refers to an absence of hydrochloric acid in the stomach. Reference:

A client is evaluated for severe pain in the right upper abdominal quadrant, which is accompanied by nausea and vomiting. The physician diagnoses acute cholecystitis and cholelithiasis. For this client, which nursing diagnosis takes top priority? Acute pain related to biliary spasms Deficient knowledge related to prevention of disease recurrence Anxiety related to unknown outcome of hospitalization Imbalanced nutrition: Less than body requirements related to biliary inflammation

Acute pain related to biliary spasms Explanation: The chief symptom of cholecystitis is abdominal pain or biliary colic. Typically, the pain is so severe that the client is restless and changes positions frequently to find relief. Therefore, the nursing diagnosis of Acute pain related to biliary spasms takes highest priority. Until the acute pain is relieved, the client can't learn about prevention, may continue to experience anxiety, and can't address nutritional concerns.

A client with liver and renal failure has severe ascites. On initial shift rounds, his primary nurse finds his indwelling urinary catheter collection bag too full to store more urine. The nurse empties more than 2,000 ml from the collection bag. One hour later, she finds the collection bag full again. The nurse notifies the physician, who suspects that a bladder rupture is allowing the drainage of peritoneal fluid. The physician orders a urinalysis to be obtained immediately. The presence of which substance is considered abnormal? Creatinine Urobilinogen Chloride Albumin

Albumin Explanation: Albumin is an abnormal finding in a routine urine specimen. Ascites present in liver failure contain albumin; therefore, if the bladder ruptured, ascites containing albumin would drain from the indwelling urinary catheter because the catheter is no longer contained in the bladder. Creatinine, urobilinogen, and chloride are normally found in urine.

Which of the following digestive enzymes aids in the digesting of starch? Amylase Lipase Trypsin Bile

Amylase Explanation: Digestive enzymes secreted by the pancreas include trypsin, which aids in digesting protein; amylase, which aids in digesting starch; and lipase, which aids in digesting fats. Bile is secreted by the liver and is not considered a digestive enzyme.

A patient is ordered desmopressin (DDAVP) for the treatment of diabetes insipidus. What therapeutic response does the nurse anticipate the patient will experience? A A decrease in blood pressure B A decrease in urine output C A decrease in appetite D A decrease in blood glucose levels

B

Patients with hyperthyroidism are characteristically: A Calm B Sensitive to heat C Apathetic and anorexic D Emotionally stable

B

The nurse is teaching a client about the dietary restrictions related to his diagnosis of hyperparathyroidism. What foods should the nurse encourage the client to avoid? A. Hamburger B. Milk C Bananas DChicken livers

B

The nurse practitioner who assesses a patient with hyperthyroidism would expect the patient to report which of the following conditions? A Fatigue B Weight loss C Hair loss D Dyspnea

B

For a client with Graves' disease, which nursing intervention promotes comfort? A. Limiting intake of high-carbohydrate foods B.Maintaining room temperature in the low-normal range C. Restricting intake of oral fluids D. Placing extra blankets on the client's bed

B Graves' disease causes signs and symptoms of hypermetabolism, such as heat intolerance, diaphoresis, excessive thirst and appetite, and weight loss.

A client is admitted to an acute care facility with a tentative diagnosis of hypoparathyroidism. The nurse should monitor the client closely for the related problem of: A excessive thirst. B profound neuromuscular irritability. C. acute gastritis. D severe hypotension.

B Hypoparathyroidism may slow bone resorption, reduce the serum calcium level, and cause profound neuromuscular irritability (as evidenced by tetany).

During an assessment of a patient with SIADH, the nurse notes the unexpected result of: A. A blood pressure reading of 120/85 mm Hg. B Pitting edema in the lower extremities. C Moist mucous membranes. D Normal skin turgor.

B In SIADH, the patient does not appear to retain fluids because reabsorbed water is intracellular rather than interstitial

A client with a history of chronic hyperparathyroidism admits to being noncompliant. Based on initial assessment findings, the nurse formulates the nursing diagnosis of Risk for injury. To complete the nursing diagnosis statement for this client, which "related-to" phrase should the nurse add? A Related to exhaustion secondary to an accelerated metabolic rate B. Related to bone demineralization resulting in pathologic fractures C. Related to tetany secondary to a decreased serum calcium level D Related to edema and dry skin secondary to fluid infiltration into the interstitial spaces

B. Poorly controlled hyperparathyroidism may cause an elevated serum calcium level. This increase, in turn, may diminish calcium stores in the bone, causing bone demineralization and setting the stage for pathologic fractures and a risk for injury

Which question will best assist the nurse in the assessment of a patient with acute diarrhea? A. "Have you had a colonoscopy in the last 3 months?" B. "Have you traveled outside the country recently?" C. "Do you have any trouble swallowing?" D. "Do you have any allergies?"

B. "Have you traveled outside the country recently?" RATIONALE A history of recent travel may help pinpoint an infectious source for the patient's diarrhea. A colonoscopy will not cause acute diarrhea. Trouble swallowing is not related to diarrhea. Allergic reactions do not typically cause acute diarrhea.

An obese male patient has sought advice from the nurse about the possible efficacy of medications in his efforts to lose weight. What should the nurse teach the patient about pharmacologic interventions for the treatment of obesity? A. "Medications are usually reserved for people who have had unsuccessful bariatric surgery." B. "Medications may be of some use, but they don't tend to resolve obesity on their own." C. "Medications are an excellent option for individuals who prefer not to exercise or reduce their food intake." D. "Medications have the potential to reduce hunger but they rarely result in weight loss."

B. "Medications may be of some use, but they don't tend to resolve obesity on their own." RATIONALE Medications for obesity rarely result in loss of more than 10% of total body weight. They are not intended as a substitute for exercise or a healthy diet. They are not solely intended for those individuals who have undergone bariatric surgery.

A patient is complaining of right lower quadrant pain, fever, and decreased appetite. What does the nurse suspect is the most likely cause? A. Diverticulitis B. Appendicitis C. Small bowel obstruction D. Sigmoid colon cancer

B. Appendicitis RATIONALE The signs and symptoms of appendicitis include right lower quadrant pain, increased fever and WBC counts, and a decreased appetite. The patient may also complain of nausea and vomiting, and the pain may radiate into the umbilical area.

A 61-year-old woman presented to a scheduled appointment with her nurse practitioner, stating, "I'm having a lot of trouble with constipation over the past few months." What action should the nurse first take in response to this patient's health complaint? A.Assess the woman's family history of constipation and bowel obstruction. B. Assess the woman's typical bowel patterns and her expectations for bowel function. C. Advise the woman to increase her fluid intake, activity level, and fiber intake. D. Arrange for a barium enema or colonoscopy to assess the woman's lower bowel.

B. Assess the woman's typical bowel patterns and her expectations for bowel function. RATIONALE Before recommending interventions, it is important to ascertain the patient's current bowel habits and her expectations surrounding these. This should precede invasive diagnostic testing and would be prioritized over the woman's family history.

A client who had a Roux-en-Y bypass procedure for morbid obesity ate a chocolate chip cookie after a meal. After ingestion of the cookie, the client reported cramping pains, dizziness, and palpitation. After having a bowel movement, the symptoms resolved. What should the nurse educate the client about regarding this event? A. Gastric outlet obstruction B. Dumping syndrome C. Bile reflux D. Celiac disease

B. Dumping syndrome RATIONALE Dumping syndrome is an unpleasant set of vasomotor and GI symptoms that occur in up to 76% of patients who have had bariatric surgery. Early symptoms include a sensation of fullness, weakness, faintness, dizziness, palpitations, diaphoresis, cramping pains, and diarrhea. These symptoms resolve once the intestine has been evacuated (i.e., with defecation).

A patient comes to the bariatric clinic to obtain information about bariatric surgery. The nurse assesses the obese patient knowing that, in addition to meeting the criterion of morbid obesity, a candidate for bariatric surgery must also demonstrate what? A. Knowledge of the causes of obesity and its associated risks B. Emotional stability and understanding of required lifestyle changes. C. Positive body image and high self-esteem D. Insight into why their past weight loss efforts failed

B. Emotional stability and understanding of required lifestyle changes. RATIONALE Patients seeking bariatric surgery must be evaluated by a psychiatrist, psychologist, or advanced practice mental health nurse to establish that they are free of serious mental disorders and are motivated to comply with lifestyle changes related to eating patterns, dietary choices, and elimination. Obese patients are often unlikely to have a positive body image due to the social stigma associated with obesity. While assessment of knowledge about causes of obesity and its associated risks, as well as insight into the reasons why previous diets have been ineffective are included in the patient's plan of care, these do not predict positive patient outcomes following bariatric surgery. Most obese patients have an impaired body image and alteration in self-esteem. An obese patient with a positive body image would be unlikely to seek this surgery unless she or he was experiencing significant comorbidities.

A client reports diarrhea after having bariatric surgery. What nonpharmacologic treatment can the nurse suggest to decrease the incidence of diarrhea? A. Decrease the fat content in the diet. B. Increase the fiber content in the diet. C. Decrease the amount of fluid the patient is drinking. D. Increase the protein content in the diet.

B. Increase the fiber content in the diet. RATIONALE Clients may complain of either diarrhea or constipation postprocedure. Diarrhea is more common an occurrence post bariatric surgery, particularly after malabsorptive procedures (Mechanick et al., 2008). Both may be prevented if the patient consumes a nutritious diet that is high in fiber. Steatorrhea also may occur as a result of rapid gastric emptying, which prevents adequate mixing with pancreatic and biliary secretions. In mild cases, reducing the intake of fat and administering an antimotility medication (e.g., loperamide [Imodium]) may control symptoms.

A patient complains of abdominal pain unrelieved by defecation, that typically occurs after meals along with diarrhea. What does the nurse recognize as the most likely diagnosis? A. Ulcerative colitis B. Regional enteritis C. Cholecystitis D. Diverticulosis

B. Regional enteritis RATIONALE Regional enteritis, or Crohn's disease, typically presents with cramp/spasm-type abdominal pain that worsens after meals. It often also presents with diarrhea.

The nurse is planning for the discharge of a client with peptic ulcer disease. Which outcome must be included in the plan of care? A. The client's pain is controlled with NSAIDs. B. The client understands and maintains lifestyle modifications. C. The client takes antacids around the clock. D. The client has no episodes of GI bleeding.

B. The client understands and maintains lifestyle modifications. RATIONALE Maintaining the lifestyle adjustments of eating an appropriate diet, reducing stress, decreasing or stopping smoking, and following a medication regimen are the goal to treat and prevent complications.

The nurse is reviewing the history and physical examination of a client diagnosed with hyperthyroidism. Which of the following would the nurse expect to find? A. Inability to tolerate cold B Thick hard nails C Reports of increased appetite D Complaints of sleepiness

C

A client is admitted to the emergency department with reports right lower quadrant pain. Blood specimens are drawn and sent to the laboratory. Which laboratory finding should be reported to the health care provider immediately? A. Hematocrit 42% B. White blood cell (WBC) count 22.8/mm3 C. Serum potassium 4.2 mEq/L D. Serum sodium 135 mEq/L

B. White blood cell (WBC) count 22.8/mm3 RATIONALE The nurse should report the elevated WBC count. This finding, which is a sign of infection, indicates that the client's appendix might have ruptured. Hematocrit of 42%, serum potassium of 4.2 mEq/L, and serum sodium of 135 mEq/L are within normal limits. Alterations in these levels don't indicate appendicitis.

A patient returns to the floor after a laparoscopic cholecystectomy. The nurse caring for the patient is aware that the most serious potential complication is what? Pulmonary atelectasis Decubitus ulcer Wound evisceration Bile duct injury

Bile duct injury Explanation: Careful screening of patients and identification of those at low risk for problems limits the frequency of conversion to an open abdominal procedure. With wider use of laparoscopic procedures, however, there may be an increase in the number of such conversions. The most serious complication after laparoscopic cholecystectomy is a bile duct injury.

Which of the following is considered the gold standard for the diagnosis of liver disease? Biopsy Paracentesis Cholecystography Ultrasonography

Biopsy Explanation: Liver biopsy is considered the gold standard for the diagnosis of liver disease. Paracentesis is the removal of fluid (ascites) from the peritoneal cavity through a puncture or a small surgical incision through the abdominal wall under sterile conditions. Cholecystography and ultrasonography may be used to detect gallstones.

A patient is brought to the emergency department by ambulance. He has hematemesis and alteration in mental status. The patient has tachycardia, cool clammy skin, and hypotension. The patient has a history of alcohol abuse. What would the nurse suspect the patient has? Hemolytic jaundice Hepatic insufficiency Bleeding esophageal varices Portal hypertension

Bleeding esophageal varices Explanation: The patient with bleeding esophageal varices may present with hematemesis, melena, or general deterioration in mental or physical status and often has a history of alcohol abuse. Signs and symptoms of shock (cool clammy skin, hypotension, tachycardia) may be present. The scenario does not describe hemolytic jaundice, hepatic insufficiency, or portal hypertension.

A client is being evaluated for hypothyroidism. During assessment, the nurse should stay alert for: A. systolic murmur at the left sternal border. B. exophthalmos and conjunctival redness. . C decreased body temperature and cold intolerance. D. flushed, warm, moist skin.

C

Beta-blockers are used in the treatment of hyperthyroidism to counteract which of the following effects? A Parasympathetic B Gastrointestinal effects C Sympathetic D Respiratory effects

C

Surgical removal of the thyroid gland is the treatment of choice for thyroid cancer. During the immediate postoperative period, the nurse knows to evaluate serum levels of __________ to assess for a serious and primary postoperative complication of thyroidectomy. A. Potassium B. Magnesium C Calcium D Sodium

C

The preferred preparation for treating hypothyroidism includes which of the following? A. Methimazole (Tapazole) B. Propylthiouracil (PTU) C. Levothyroxine (Synthroid) D. Radioactive iodine

C

What is the most common cause of hyperaldosteronism? A. A pituitary adenoma B. Deficient potassium intake C. An adrenal adenoma D. Excessive sodium intake

C

What life-threatening outcome should the nurse monitor for in a client who is not compliant with taking his antithyroid medication? A Diabetes insipidus B Syndrome of inappropriate antidiuretic hormone secretion C Thyrotoxic crisis DMyxedema coma

C

Which outcome indicates that treatment of a client with diabetes insipidus has been effective? A. Urine output measures more than 200 ml/hour. B Heart rate is 126 beats/minute. C Fluid intake is less than 2,500 ml/day. D Blood pressure is 90/50 mm Hg.

C

While assessing a client with hypoparathyroidism, the nurse taps the client's facial nerve and observes twitching of the mouth and tightening of the jaw. The nurse would document this finding as which of the following? A Positive Trousseau's sign B Tetany C Positive Chvostek's sign D Hyperactive deep tendon reflex

C

After a thyroidectomy, the client develops a carpopedal spasm while the nurse is taking a BP reading on the left arm. Which action by the nurse is appropriate? A. Administer a sedative as ordered. B. Administer an oral calcium supplement as ordered. C Administer IV calcium gluconate as ordered. D Start administering oxygen at 2 L/min via a cannula.

C When hypocalcemia and tetany occur after a thyroidectomy, the immediate treatment is administration of IV calcium gluconate. If this does not immediately decrease neuromuscular irritability and seizure activity, sedative agents such as pentobarbital may be administered

What is the most common cause of small-bowel obstruction? A. Hernias B. Neoplasms C. Adhesions D. Volvulus

C. Adhesions RATIONALE Adhesions are scar tissue that forms as a result of inflammation and infection. Adhesions are the most common cause of small-bowel obstruction, followed by tumors, Crohn's disease, and hernias. Other causes include intussusception, volvulus, and paralytic ileus.

A client is admitted to the health care facility with a diagnosis of a bleeding gastric ulcer. The nurse expects the client's stools to have which description? A. Coffee-ground-like B. Clay-colored C. Black and tarry D. Bright red

C. Black and tarry RATIONALE Black, tarry stools are a sign of bleeding high in the GI tract, as from a gastric ulcer, and result from the action of digestive enzymes on the blood. Vomitus associated with upper GI tract bleeding commonly is described as coffee-ground-like. Clay-colored stools are associated with biliary obstruction. Bright red stools indicate lower GI tract bleeding.

The nurse is assessing a client with advanced gastric cancer. The nurse anticipates that the assessment will reveal which finding? A. Abdominal pain below the umbilicus B. Weight gain C. Bloating after meals D. Increased appetite

C. Bloating after meals RATIONALE Symptoms of progressive disease include bloating after meals, weight loss, abdominal pain above the umbilicus, loss or decrease in appetite, and nausea or vomiting.

A patient returns to his room following a diagnostic colonoscopy after radiologic evidence of diverticulosis. He reports an increase in abdominal pain, fever, and chills. Which clinical condition is most concerning to the nurse? A. Colon cancer B. Hemorrhoids C. Bowel perforation D. Anal fissure

C. Bowel perforation RATIONALE During a colonoscopy, manipulation of the bowel occurs, which can cause peritonitis or bowel perforation. Signs and symptoms of a perforated bowel include abdominal pain, fever, and chills. The patient may have guarding (not allowing palpation of the abdomen), rigidity, and firmness of the abdomen, or rebound tenderness (an increase of pain upon releasing pressure on the abdomen).

A patient complains of abdominal pain and distention, fever, tachycardia, and diaphoresis. An abdominal x-ray shows free air under the diaphragm. The emergency department nurse should suspect which condition? A. Intestinal obstruction B. Malabsorption C. Intestinal perforation D. Acute cholelithiasis

C. Intestinal perforation RATIONALE Free air in the abdomen may result from a perforation of the abdominal organ or any part of the bowel, a tumor, or trauma.

The nurse is performing detailed patient education with a 40-year-old woman who will be soon discharged following a Roux-en-Y gastric bypass. The nurse and other members of the interdisciplinary team have been emphasizing the need for eating small amounts of food at a sitting and eating food slowly. What is the rationale for the nurse's advice? A. Eating too quickly can cause gastric ulceration. B. The cardiac sphincter is unable to dilate quickly after bariatric surgery. C. Nausea and esophageal distention can result from eating too fast. D. Eating quickly is associated with weight gain.

C. Nausea and esophageal distention can result from eating too fast. RATIONALE Because of the physical alterations to the upper gastrointestinal (GI) tract that are created during bariatric surgery, the patient is prone to nausea and esophageal distention if he or she eats too quickly. Eating quickly does not contribute to ulceration or weight gain following surgery. The cardiac sphincter is not modified during bariatric surgery.

What assessment finding supports a client's diagnosis of gastric ulcer? A. Presence of blood in the client's stool for the past month B. Complaints of sharp pain in the abdomen after eating a heavy meal C. Periods of pain shortly after eating any food. D. Complaints of epigastric burning that moves like a wave

C. Periods of pain shortly after eating any food. RATIONALE Experiencing sharp pain 30 to 60 minutes after meals is common with gastric ulcers; patients with duodenal ulcers can have night pain that is relieved by eating.

A patient has a bowel perforation from a recent surgery and now has been diagnosed with peritonitis. The patient has hypoactive bowel sounds, a temperature of 100.5°F, and an elevated WBC count. What is the most serious potential complication of peritonitis for which the nurse should monitor? A. Nausea B. Diarrhea C. Sepsis D. Abdominal tenderness

C. Sepsis RATIONALE Sepsis is the major cause of death from peritonitis. Shock may result from septicemia or hypovolemia. The inflammatory process may cause intestinal obstruction, primarily from the development of bowel adhesions, and care must be made to closely monitor the patient's nasogastric tube drainage and input and output. Diarrhea would normally not occur due to intestinal obstruction, which causes decreased peristalsis and decreased bowel movements. Nausea and abdominal tenderness are expected as clinical manifestations of peritonitis.

While assessing a client with Cushing syndrome, the nurse should expect high blood glucose reading due to increased secretion of which of the following? A. The thyroid gland B. The parathyroid glands C. The adrenal glands D. The pituitary gland

C. The adrenal glands RATIONALE In Cushing's syndrome, increased secretion of glucocorticoid hormones from the adrenal glands causes increased levels of blood glucose.

A 56-year-old presented to her nurse practitioner because she had been experiencing unprecedented constipation and the passage of pencil-like stools despite her high fluid and fiber intake. The nurse recognized the need to assess the patient for colorectal cancer and ordered diagnostic evaluations. What component of the patient's blood work would be most indicative of the presence of cancer? C-reactive protein (CRP) Carcinoembryonic antigen (CEA) Ceruloplasmin Coproporphyrin

Carcinoembryonic antigen (CEA) Explanation: CEA is a protein that is normally not detected in the blood of a healthy person; therefore, when detected it indicates that cancer is present. The other cited blood analyses are not associated with cancer.

Which foods should be avoided following acute gallbladder inflammation? Cooked fruits Cheese Coffee Mashed potatoes

Cheese Explanation: The client should avoid eggs, cream, pork, fried foods, cheese, rich dressings, gas-forming vegetables, and alcohol. It is important to remind the client that fatty foods may induce an episode of cholecystitis. Cooked fruits, rice or tapioca, lean meats, mashed potatoes, non-gas-forming vegetables, bread, coffee, or tea may be consumed as tolerated.

A very ill client with acute cholecystitis is scheduled for surgery. The surgeon plans to create an incision in the common bile duct to remove stones. The nurse correctly documents this surgery in the electronic medical record using which term? Choledochostomy Cholecystostomy Cholecystectomy Choledochoduodenostomy

Choledochostomy Explanation: This procedure is reserved for the client with acute cholecystitis who may be too ill to undergo a surgical procedure. It involves making an incision in the common duct, usually to remove stones (choledochostomy). REMEMBER: CHOLE-DOCH-OSTOMY "CHOLE"=CHOLECYSTITIS "DOCH" SOUNDS LIKE COMMON "DUCT" "-OSTOMY"=SURGICAL OPENING

A client with calculi in the gallbladder is said to have Cholecystitis Cholelithiasis Choledocholithiasis Choledochotomy

Cholelithiasis Explanation: Calculi, or gallstones, usually form in the gallbladder from the solid constituents of bile; they vary greatly in size, shape, and composition. Cholecystitis is acute inflammation of the gallbladder. Choledocholithiasis is a gallstone in the common bile duct. Choledochotomy is an incision into the common bile duct.

When assessing a client with cirrhosis of the liver, which of the following stool characteristics is the client likely to report? Yellow-green Black and tarry Blood tinged Clay-colored or whitish

Clay-colored or whitish Explanation: Many clients report passing clay-colored or whitish stools as a result of no bile in the gastrointestinal tract. The other stool colors would not be absolute indicators of cirrhosis of the liver but may indicate other GI tract disorders.

A nurse is caring for a client with cholelithiasis. Which sign indicates obstructive jaundice? Straw-colored urine Reduced hematocrit Clay-colored stools Elevated urobilinogen in the urine

Clay-colored stools Explanation: Obstructive jaundice develops when a stone obstructs the flow of bile in the common bile duct. When the flow of bile to the duodenum is blocked, the lack of bile pigments results in a clay-colored stool. In obstructive jaundice, urine tends to be dark amber (not straw-colored) as a result of soluble bilirubin in the urine. Hematocrit levels aren't affected by obstructive jaundice. Because obstructive jaundice prevents bilirubin from reaching the intestine (where it's converted to urobilinogen), the urine contains no urobilinogen.

A client is scheduled for several diagnostic tests to evaluate gastrointestinal function. After teaching the client about these tests, the nurse determines that the client has understood the teaching when the client identifies which test as not requiring the use of a contrast medium? Small bowel series Computer tomography Colonoscopy Upper GI series

Colonoscopy Explanation: A colonoscopy is a direct visual examination of the entire large intestine. It does not involve the use of a contrast agent. Contrast medium may be used with a small bowel series, computed tomography, and upper GI series.

A nurse is planning care for a client in acute addisonian crisis. Which nursing diagnosis should receive the highest priority? A Risk for infection B Imbalanced nutrition: Less than body requirements C Impaired physical mobility D Decreased cardiac output

D

An adult patient has undergone extensive testing that has resulted in a diagnosis of a basophilic pituitary tumor. The pathophysiological effects of the patient's tumor include excessive secretion of adrenocorticotropic hormone (ACTH). As a result, this patient is likely to exhibit signs and symptoms that are characteristic of what endocrine disorder? A. Addison's disease B. Hyperthyroidism C. Diabetes insipidus D. Cushing's disease

D

The nurse is closely monitoring the blood work of a patient who has a diagnosis of primary hyperparathyroidism. The nurse should be aware that the fluid and electrolyte disturbances associated with this disease create a significant risk of what problems? A. Metabolic acidosis and cardiac ischemia B Fluid volume overload and pruritus C Deep vein thrombosis and pulmonary embolism D. Renal calculi and urinary obstructio

D

Which nursing diagnosis takes highest priority for a client with hyperthyroidism? A. Disturbed body image related to weight gain and edema B. Risk for imbalanced nutrition: More than body requirements related to thyroid hormone excess C. Risk for impaired skin integrity related to edema, skin fragility, and poor wound healing D. imbalanced nutrition: Less than body requirements related to thyroid hormone excess

D

A nurse should expect a client with hypothyroidism to report: A. thyroid gland swelling. B. nervousness and tremors. C. increased appetite and weight loss. D puffiness of the face and hands.

D Hypothyroidism (myxedema) causes facial puffiness, extremity edema, and weight gain. Signs and symptoms of hyperthyroidism (Graves' disease) include an increased appetite, weight loss, nervousness, tremors, and thyroid gland enlargement (goiter).

Which of the following endocrine disorder causes the patient to have dilutional hyponatremia? A Diabetes insipidus (DI) B Hyperthyroidism C Hypothyroidism D Syndrome of inappropriate antidiuretic hormone secretion (SIADH)

D Patients diagnosed with SIADH retain water and develop a subsequent sodium deficiency known as dilutional hyponatremia. In DI, there is excessive thirst and large volumes of dilute urine. Patients with DI, hypothyroidism, or hyperthyroidism do not exhibit dilutional hyponatremia

A community health nurse is performing a home visit to a 53-year-old patient who requires twice-weekly wound care on her foot. The patient mentions that she is currently having hemorrhoids, a problem that she has not previously experienced. What treatment measure should the nurse recommend to this patient? A. Daily application of topical antibiotics B. Decreased fluid intake C. Bathing, rather than showering, once per day D. A high-fiber diet with increased fruit intake

D. A high-fiber diet with increased fruit intake RATIONALE Hemorrhoid symptoms and discomfort can be relieved by good personal hygiene and by avoiding excessive straining during defecation. A high-residue diet that contains fruit and bran along with an increased fluid intake may be all the treatment necessary to promote the passage of soft, bulky stools to prevent straining. It is unnecessary to avoid showering, and antibiotics are not an effective treatment.

When caring for a client with an acute exacerbation of a peptic ulcer, the nurse finds the client doubled up in bed with severe pain in the right shoulder. What is the intial appropriate action by the nurse? A. Notify the health care provider. B. Irrigate the client's NG tube. C. Place the client in the high- Fowler's position. D. Assess the client's abdomen and vital signs.

D. Assess the client's abdomen and vital signs. RATIONALE Signs and symptoms of perforation includes sudden, severe upper abdominal pain (persisting and increasing in intensity); pain may be referred to the shoulders, especially the right shoulder, because of irritation of the phrenic nerve in the diaphragm. The nurse should assess the vital signs and abdomen prior to notifying the physician. Irrigation of the NG tube should not be performed because the additional fluid may be spilled into the peritoneal cavity, and the client should be placed in a position of comfort, usually on the side with the head slightly elevated.

When caring for a client with an acute exacerbation of a peptic ulcer, the nurse finds the client doubled up in bed with severe pain in the right shoulder. What is the intial appropriate action by the nurse? A. Notify the health care provider. B. Irrigate the client's NG tube. C. Place the client in the high-Fowler's position. D. Assess the client's abdomen and vital signs.

D. Assess the client's abdomen and vital signs. RATIONALE Signs and symptoms of perforation includes sudden, severe upper abdominal pain (persisting and increasing in intensity); pain may be referred to the shoulders, especially the right shoulder, because of irritation of the phrenic nerve in the diaphragm. The nurse should assess the vital signs and abdomen prior to notifying the physician. Irrigation of the NG tube should not be performed because the additional fluid may be spilled into the peritoneal cavity, and the client should be placed in a position of comfort, usually on the side with the head slightly elevated.

Which of the following clients is at highest risk for peptic ulcer disease? A. Client with blood type A B. Client with blood type B C. Client with blood type AB D. Client with blood type O

D. Client with blood type O RATIONALE Clients with blood type O are more susceptible to peptic ulcers than those with blood types A, B, and AB.

A patient with pheochromocytoma has been admitted for an adrenalectomy tomorrow. The patient is to start IV medication this evening to prevent adrenal insufficiency. What medication is the patient most likely to require? A. Antibiotics B. Antihypertensives C. Parenteral nutrition D. Corticosteroids

D. Corticosteroids RATIONALE The adrenal cortex produces corticosteroids. As a result, corticosteroids would be administered to prevent adrenal insufficiency. Antibiotics, antihypertensives, and parenteral nutrition do not prevent adrenal insufficiency.

Which assessment finding does the nurse expect as a normal consequence of aging? A. Increased salivation and drooling B. Hyperactive bowel sounds and loose stools C. Increased gastric production and heartburn D. Decreased sensation to defecate and constipation

D. Decreased sensation to defecate and constipation RATIONALE Older adults may lose the sensation to defecate, resulting in constipation. Salivation decreases with aging, along with peristalsis and gastric acid production.

An 81-year-old patient of a subacute geriatric medical unit has become incontinent of stool over the past several days. This development has coincided with a decline in the patient's cognition as a result of severe Alzheimer's disease. In light of the patient's fecal incontinence, what nursing diagnosis should the nurse prioritize when planning this patient's care? A. Altered role performance related to fecal incontinence B. Altered nutrition, less than body requirements related to fecal incontinence C. Situational low self-esteem related to fecal incontinence D. Impaired skin integrity related to fecal incontinence

D. Impaired skin integrity related to fecal incontinence RATIONALE Fecal incontinence creates a significant risk of skin breakdown, especially when combined with decreased cognition. Role performance and self-esteem are likely to be impacted by the patient's Alzheimer's disease more than by fecal incontinence. Incontinence does not necessarily cause a nutritional deficit.

The client with a peptic ulcer is admitted to the hospital's intensive care unit with obvious gastric bleeding. What is the priority intervention for the nurse? A. Keep an accurate record of intake and output. B. Provide for quiet environment, restrict visitors. C. Prepare the client for an endoscopy. D. Monitor vital signs and observe for signs of hypovolemia.

D. Monitor vital signs and observe for signs of hypovolemia. RATIONALE The goal is to directly stop the bleeding and remove blood/clots/secretions from GI tract so that an endoscopy can be performed and the patient does not vomit and aspirate gastric contents.

When performing an abdominal assessment on a patient with suspected cholecystitis, how does the nurse palpate the patient's abdomen? A. Palpate the right lower quadrant only B. Palpate the upper quadrants only C. Defer palpation and use percussion only D. Palpate the right upper quadrant last

D. Palpate the right upper quadrant last RATIONALE The patient with cholecystitis will report pain in the right upper quadrant of the abdomen. Tender or painful areas should be palpated last to prevent the patient from tensing his or her abdominal muscles because of pain, thereby making the examination more difficult. Palpation is an important assessment tool that should not be deferred for this patient.

A client is in the hospital for the treatment of peptic ulcer disease. The client reports vomiting and a sudden severe pain in the abdomen. The nurse then assesses a board-like abdomen. What does the nurse suspect these symptoms indicate? A. Ineffective treatment for the peptic ulcer B. A reaction to the medication given for the ulcer C. Gastric penetration D. Perforation of the peptic ulcer

D. Perforation of the peptic ulcer RATIONALE Signs and symptoms of perforation include the following: Sudden, severe upper abdominal pain (persisting and increasing in intensity), which may be referred to the shoulders, especially the right shoulder, because of irritation of the phrenic nerve in the diaphragm; vomiting; collapse (fainting); extremely tender and rigid (boardlike) abdomen; and hypotension and tachycardia, indicating shock.

A client is recovering from gastric surgery. Toward what goal should the nurse progress the client's enteral intake? A. Three meals and 120 ml fluid daily B. Three meals and three snacks and 120 mL fluid daily C. Six small meals and 120 mL fluid daily D. Six small meals daily with 120 mL fluid between meals

D. Six small meals daily with 120 mL fluid between meals RATIONALE After the return of bowel sounds and removal of the nasogastric tube, the nurse may give fluids, followed by food in small portions. Foods are gradually added until the client can eat six small meals a day and drink 120 mL of fluid between meals. Reference:

The nurse recognizes that most nutrients and electrolytes are absorbed by which organ? A. Esophagus B. Stomach C. Colon D. Small intestine

D. Small intestine RATIONALE The small intestine absorbs most of the nutrients and electrolytes. The colon absorbs water, sodium, and chloride from the digested food that has passed from the small intestine. The esophagus moves food from the mouth to the stomach, which stores food during eating and secretes digestive fluids.

A client informs the nurse that he is taking a stimulant laxative in order to be able to have a bowel movement daily. What should the nurse inform the client about taking a stimulant laxative? A. As long as the client is drinking 8 glasses of water per day, he can continue to take them. B. The laxative is safe to take with other medication the client is taking. C. The client should take a fiber supplement along with the stimulant laxative. D. They can be habit forming and will require increasing doses to be effective.

D. They can be habit forming and will require increasing doses to be effective. RATIONALE The nurse should discourage self-treatment with daily or frequent enemas or laxatives. Chronic use of such products causes natural bowel function to be sluggish. In addition, laxatives continuing stimulants can be habit forming, requiring continued use in increasing doses. Although the nurse should encourage the client to have adequate fluid intake, laxative use should not be encouraged. The laxative may interact with other medications the client is taking and may cause a decrease in absorption. A fiber supplement may be taken alone but should not be taken with a stimulant laxative.

A 70-year-old client is admitted with acute pancreatitis. The nurse understands that the mortality rate associated with acute pancreatitis increases with advanced age and attributes this to which gerontologic consideration associated with the pancreas? Decreases in the physiologic function of major organs Increases in the bicarbonate output by the kidneys Increases in the rate of pancreatic secretion Development of local complications

Decreases in the physiologic function of major organs Explanation: Acute pancreatitis affects people of all ages, but the mortality rate associated with acute pancreatitis increases with advancing age. The pattern of complications changes with age. Younger clients tend to develop local complications; the incidence of multiple organ failure increases with age, possibly as a result of progressive decreases in physiologic function of major organs with increasing age.

A client is receiving vasopressin for the urgent management of active bleeding due to esophageal varices. What most serious complication should the nurse assess the client for after the administration? Urinary output changes Electrocardiogram changes Electrolytes level changes Liver enzyme changes

Electrocardiogram changes Explanation: Vasopressin (Pitressin) is administered during the management of an urgent situation with an acute esophageal bleed because of its vasoconstrictive properties in the splanchnic, portal, and intrahepatic vessels. This medication also causes coronary artery constriction that may dispose clients with coronary artery disease to cardiac ischemia; therefore, the nurse observes the client for evidence of chest pain, ECG changes, and vital sign changes. Vasopressin will does not infer with urinary output, electrolytes, or liver enzymes.

A client has an elevated serum ammonia concentration and is exhibiting changes in mental status. The nurse should suspect which condition? Hepatic encephalopathy Portal hypertension Asterixis Cirrhosis

Hepatic encephalopathy Explanation: Hepatic encephalopathy is a central nervous system dysfunction resulting from liver disease. It is frequently associated with an elevated ammonia concentration that produces changes in mental status, altered level of consciousness, and coma. Portal hypertension is an elevated pressure in the portal circulation resulting from obstruction of venous flow into and through the liver. Asterixis is an involuntary flapping movement of the hands associated with metabolic liver dysfunction.

When reviewing the history of a client with pancreatic cancer, the nurse would identify which of the following as a possible risk factor? History of pancreatitis Ingestion of a low-fat diet One-time exposure to petrochemicals Ingestion of caffeinated coffee

History of pancreatitis Explanation: Pancreatitis is associated with the development of pancreatic cancer. Other factors that correlate with pancreatic cancer include diabetes mellitus, a high-fat diet, and chronic exposure to carcinogenic substances (i.e., petrochemicals). Although data are inconclusive, a relationship may exist between cigarette smoking and high coffee consumption (especially decaffeinated coffee) and the development of pancreatic carcinoma.

A client is being prepared to undergo laboratory and diagnostic testing to confirm the diagnosis of cirrhosis. Which test would the nurse expect to be used to provide definitive confirmation of the disorder? Coagulation studies Magnetic resonance imaging Radioisotope liver scan Liver biopsy

Liver biopsy Explanation: A liver biopsy which reveals hepatic fibrosis is the most conclusive diagnostic procedure. Coagulation studies provide information about liver function but do not definitively confirm the diagnosis of cirrhosis. Magnetic resonance imaging and radioisotope liver scan help to support the diagnosis but do not confirm it. These tests provide information about the liver's enlarged size, nodular configuration, and distorted blood flow.

A physician orders spironolactone (Aldactone), 50 mg by mouth four times daily, for a client with fluid retention caused by cirrhosis. Which finding indicates that the drug is producing a therapeutic effect? Serum potassium level of 3.5 mEq/L Loss of 2.2 lb (1 kg) in 24 hours Serum sodium level of 135 mEq/L Blood pH of 7.25

Loss of 2.2 lb (1 kg) in 24 hours Explanation: Daily weight measurement is the most accurate indicator of fluid status; a loss of 2.2 lb (1 kg) indicates loss of 1 L of fluid. Because spironolactone is a diuretic, weight loss is the best indicator of its effectiveness. This client's serum potassium and sodium levels are normal. A blood pH of 7.25 indicates acidosis, an adverse reaction to spironolactone.

A nurse is planning care for a client with acute pancreatitis. Which client outcome does the nurse assign as the highest priority? Developing no acute complications from the pancreatitis Maintaining normal respiratory function Maintaining satisfactory pain control Achieving adequate fluid and electrolyte balance

Maintaining normal respiratory function Explanation: Airway and breathing are always the priority assessment. Acute pancreatitis produces retroperitoneal edema, elevation of the diaphragm, pleural effusion, and inadequate lung ventilation. Intra-abdominal infection and labored breathing increase the body's metabolic demands, which further decreases pulmonary reserve and can lead to respiratory failure. Maintenance of adequate respiratory function is the priority goal. The other outcomes would also be appropriate for the patient.

A nurse is giving a client barium swallow test. What is the most important assessment a nurse would make to ensure that a client does not retain any barium after a barium swallow? Placing any stool passed in a specific preservative. Monitoring the stool passage and its color. Observing the color of urine. Monitoring the volume of urine.

Monitoring the stool passage and its color. Explanation: Monitoring stool passage and its color will ensure that the client remains barium free following a barium swallow test. The white or clay color of the stool would indicate barium retention. The stool should be placed in a special preservative if the client undergoes a stool analysis. Observing the color and volume of urine will not ensure that the client is barium free because barium is not eliminated through urine but through stool

The nurse identifies a potential collaborative problem of electrolyte imbalance for a client with severe acute pancreatitis. Which assessment finding alerts the nurse to an electrolyte imbalance associated with acute pancreatitis? Muscle twitching and finger numbness Paralytic ileus and abdominal distention Hypotension Elevated blood glucose concentration

Muscle twitching and finger numbness Explanation: Muscle twitching and finger numbness indicate hypocalcemia, a potential complication of acute pancreatitis. Calcium may be prescribed to prevent or treat tetany, which may result from calcium losses into retroperitoneal (peripancreatic) exudate. The other data indicate other complications of acute pancreatitis but are not indicators of electrolyte imbalance.

A client is actively bleeding from esophageal varices. Which medication would the nurse most expect to be administered to this client? Octreotide Spironolactone Propranolol Lactulose

Octreotide Explanation: In an actively bleeding client, medications are administered initially because they can be obtained and administered quicker than other therapies. Octreotide (Sandostatin) causes selective splanchnic vasoconstriction by inhibiting glucagon release and is used mainly in the management of active hemorrhage. Propranolol (Inderal) and nadolol (Corgard), beta-blocking agents that decrease portal pressure, are the most common medications used both to prevent a first bleeding episode in clients with known varices and to prevent rebleeding. Beta-blockers should not be used in acute variceal hemorrhage, but they are effective prophylaxis against such an episode. Spironolactone (Aldactone), an aldosterone-blocking agent, is most often the first-line therapy in clients with ascites from cirrhosis. Lactulose (Cephulac) is administered to reduce serum ammonia levels in clients with hepatic encephalopathy.

A nurse practitioner examined a patient who had been diagnosed with hepatomegaly (enlarged liver) due to accumulated fat deposits in the liver, subsequent to obesity. The nurse would palpate the liver by placing: One hand under the left lower rib cage and pressing upward toward the midline. Both hands over the left lower quadrant and applying gentle pressure. The left hand at the level of the umbilicus and the right hand at the base of the diaphragm. One hand under the right lower rib cage and press downward with the other hand.

One hand under the right lower rib cage and press downward with the other hand. Explanation: Refer to Figure 21-8 in the text for an illustration of this procedure. The liver is located under the diaphragm on the right side of the abdominal cavity, extending slightly left from the midline.

The nurse is teaching a client who was admitted to the hospital with acute hepatic encephalopathy and ascites about an appropriate diet. The nurse determines that the teaching has been effective when the client chooses which food choice from the menu? Omelet with green peppers, onions, mushrooms, and cheese with milk Pancakes with butter and honey, and orange juice Ham and cheese sandwich, baked beans, potatoes, and coffee Baked chicken with sweet potato french fries, cornbread, and tea

Pancakes with butter and honey, and orange juice Explanation: Teach clients to select a diet high in carbohydrates with protein intake consistent with liver function. The client should identify foods high in carbohydrates and within protein requirements (moderate to high protein in cirrhosis and hepatitis, low protein in hepatic failure). The client with acute hepatic encephalopathy is placed on a low-protein diet to decrease ammonia concentration. The other choices are all higher in protein. The client's ascites indicates that a low-sodium diet is needed, and the other choices are all high in sodium.

A 37-year-old male patient presents at the emergency department complaining of nausea and vomiting and severe abdominal pain. While the nurse is assessing the patient, the patient's wife informs the nurse that the patient ingested 24 ounces of vodka last evening. The patient's abdomen is rigid, and there is bruising to the patient's flank. What is the patient exhibiting signs of? Pancreatitis with possible peritonitis Acute cholecystitis Obstruction of the bowel Acute appendicitis

Pancreatitis with possible peritonitis Explanation: Severe abdominal pain is the major symptom of pancreatitis that causes the patient to seek medical care. Pain in pancreatitis is accompanied by nausea and vomiting that does not relieve the pain or nausea. Abdominal guarding is present, and a rigid or board-like abdomen may be a sign of peritonitis. Ecchymosis (bruising) to the flank or around the umbilicus may indicate severe peritonitis. Pain generally occurs 24 to 48 hours after a heavy meal or alcohol ingestion.

Which of the following is an enzyme secreted by the gastric mucosa? Pepsin Trypsin Ptyalin Bile

Pepsin Explanation: Pepsin is secreted by the gastric mucosa. Trypsin is secreted by the pancreas. The salivary glands secrete ptyalin. The liver and gallbladder secrete bile.

A client undergoing a diagnostic examination for gastrointestinal disorder was given polyethylene glycol/electrolyte solution as a part of the test preparation. Which of the following measures should the nurse take once the solution is administered? Instruct the client to have low-residue meals. Allow the client to ingest fat-free meal. Permit the client to drink only clear liquids. Provide saline gargles to the client.

Permit the client to drink only clear liquids. Explanation: After polyethylene glycol/electrolyte solution is administered, the client should have clear liquids because this ensures watery stools, which are necessary for procedures like a barium enema. Allowing the client to ingest a fat-free meal is used in preparation for oral cholecystography. Instructing the client to have low-residue meals is a pretest procedure for barium enema. A client is offered saline gargles after esophagogastroduodenoscopy.

Which is the most common cause of esophageal varices? Jaundice Portal hypertension Ascites Asterixis

Portal hypertension Explanation: Esophageal varices are almost always caused by portal hypertension, which results from obstruction of the portal circulation within the damaged liver. Jaundice occurs when the bilirubin concentration in the blood is abnormally elevated. Ascites results from circulatory changes within the diseased liver. Asterixis is an involuntary flapping movement of the hands associated with metabolic liver dysfunction.

When caring for a client with acute pancreatitis, the nurse should use which comfort measure? Administering an analgesic once per shift, as ordered, to prevent drug addiction Positioning the client on the side with the knees flexed Encouraging frequent visits from family and friends Administering frequent oral feedings

Positioning the client on the side with the knees flexed Explanation: The nurse should place the client with acute pancreatitis in a side-lying position with knees flexed; this position promotes comfort by decreasing pressure on the abdominal muscles. The nurse should administer an analgesic, as needed and ordered, before pain becomes severe, rather than once each shift. Because the client needs a quiet, restful environment during the acute disease stage, the nurse should discourage frequent visits from family and friends. Frequent oral feedings are contraindicated during the acute stage to allow the pancreas to rest.

A client being treated for pancreatitis faces the risk of atelectasis. Which of the following interventions would be important to implement to minimize this risk? Monitor pulse oximetry every hour. Withhold oral feedings for the client. Instruct the client to avoid coughing. Reposition the client every 2 hours.

Reposition the client every 2 hours. Explanation: Repositioning the client every 2 hours minimizes the risk of atelectasis in a client who is being treated for pancreatitis. The client should be instructed to cough every 2 hours to reduce atelectasis. Monitoring the pulse oximetry helps show changes in respiratory status and promote early intervention, but it would do little to minimize the risk of atelectasis. Withholding oral feedings limits the reflux of bile and duodenal contents into the pancreatic duct.

The nurse is providing care to a patient with gross ascites who is maintaining a position of comfort in the high semi-Fowler's position. What is the nurse's priority assessment of this patient? Respiratory assessment related to increased thoracic pressure Urinary output related to increased sodium retention Peripheral vascular assessment related to immobility Skin assessment related to increase in bile salts

Respiratory assessment related to increased thoracic pressure Explanation: If a patient with ascites from liver dysfunction is hospitalized, nursing measures include assessment and documentation of intake and output (I&O;), abdominal girth, and daily weight to assess fluid status. The nurse also closely monitors the respiratory status because large volumes of ascites can compress the thoracic cavity and inhibit adequate lung expansion. The nurse monitors serum ammonia, creatinine, and electrolyte levels to assess electrolyte balance, response to therapy, and indications of encephalopathy.

A patient who had surgery for gallbladder disease has just returned to the unit. The nurse caring for this patient knows to immediately report what assessment finding to the primary care provider? Decreased breath sounds Drainage of "bile-colored fluid" onto the abdominal dressing Rigidity of the abdomen Acute pain with movement

Rigidity of the abdomen Explanation: The location of the subcostal incision will likely cause the patient to take shallow breaths to prevent pain, and this may result in decreased breath sounds. The nurse should remind patients to take deep breaths and cough to expand the lungs fully and prevent atelectasis. Acute pain is an expected assessment finding following surgery, and analgesics should be administered for pain relief. Abdominal splinting or application of an abdominal binder may assist in reducing the pain. Bile may continue to drain from the drainage tract after surgery, and this will require frequent changes of the abdominal dressing. Increased abdominal tenderness and rigidity should be reported immediately to the health care provider, as it may indicate bleeding from an inadvertent puncture or nicking of a major blood vessel during the surgical procedure.

A patient has been admitted to the critical care unit from the subacute medical unit because his signs and symptoms of liver failure have become more pronounced over the past 24 hours. The critical care nurse who is planning this patient's care should prioritize which of the following nursing diagnoses? Risk for bleeding related to complications of liver failure Knowledge deficit related to the causes of liver failure Bowel incontinence related to treatments for liver failure Risk for impaired gas exchange related to complications of liver failure

Risk for bleeding related to complications of liver failure Explanation: Hematological changes and the possibility of esophageal varices present an acute risk of hemorrhage in the patient with liver failure. This risk supersedes the possibility of respiratory complications. Knowledge deficit and bowel incontinence are likely, but are not more important than the possibility of hemorrhage.

Over the past 2 years, a 51-year-old man has been admitted to the hospital five times for problems related to alcohol abuse, including falls, acute alcohol withdrawal, and cirrhosis. The man's current admission has been precipitated by signs and symptoms of alcoholic liver disease (ALD). The care team is reasonably pleased with the patient's recovery to this point and discharge planning has begun. When organizing the patient's discharge planning, what psychosocial nursing diagnosis should be prioritized? Risk for altered family processes Risk for ineffective coping Risk for ineffective management of therapeutic regimen Risk for altered growth and development

Risk for ineffective management of therapeutic regimen Explanation: Abstinence from alcohol use is critical for patients who are to recover from ALD; a failure to adhere to this aspect of the therapeutic regimen is likely to result in death. As a result, this is prioritized over family processes, growth and development, and coping, even though each of these issues is likely relevant to the patient's circumstances.

Clients with chronic liver dysfunction have problems with insufficient vitamin intake. Which may occur as a result of vitamin C deficiency? Night blindness Hypoprothrombinemia Scurvy Beriberi

Scurvy Explanation: Scurvy may result from a vitamin C deficiency. Night blindness, hypoprothrombinemia, and beriberi do not result from a vitamin C deficiency.

A client comes to the ED with severe abdominal pain, nausea, and vomiting. The physician plans to rule out acute pancreatitis. The nurse would expect the diagnosis to be confirmed by an elevated result on which laboratory test? Serum calcium Serum bilirubin Serum amylase Serum potassium

Serum amylase Explanation: Serum amylase and lipase concentrations are used to make the diagnosis of acute pancreatitis. Serum amylase and lipase concentrations are elevated within 24 hours of the onset of symptoms. Serum amylase usually returns to normal within 48 to 72 hours, but the serum lipase concentration may remain elevated for a longer period, often days longer than amylase. Urinary amylase concentrations also become elevated and remain elevated longer than serum amylase concentrations.

The nurse is instructing the client who was newly diagnosed with peptic ulcers. Which of the following diagnostic studies would the nurse anticipate reviewing with the client? A complete blood count including differential Serum antibodies for H. pylori A sigmoidoscopy Gastric analysis

Serum antibodies for H. pylori Explanation: Helicobacter pylori, a bacterium, is believed to be responsible for the majority of peptic ulcers. Blood tests are used to determine whether there are antibodies to H. pylori in the blood. A complete blood count with differential can indicate bleeding and infection associated with a bleeding ulcer. A sigmoidoscopy assesses the lower gastrointestinal tract. Gastric analysis is more common in analyzing gastric fluid in determining problems with the secretory activity of the gastric mucosa.

The nurse is admitting a patient to the intensive care unit with a diagnosis of acute pancreatitis. What does the nurse expect was the reason the patient came to the hospital? Severe abdominal pain Fever Jaundice Mental agitation

Severe abdominal pain Explanation: Severe abdominal pain is the major symptom of pancreatitis that causes the patient to seek medical care. Abdominal pain and tenderness and back pain result from irritation and edema of the inflamed pancreas.

A nurse is caring for a client with cirrhosis. The nurse assesses the client at noon and discovers that the client is difficult to arouse and has an elevated serum ammonia level. The nurse should suspect which situation? The client's hepatic function is decreasing. The client didn't take his morning dose of lactulose (Cephulac). The client is relaxed and not in pain. The client is avoiding the nurse.

The client's hepatic function is decreasing. Explanation: The decreased level of consciousness caused by an increased serum ammonia level indicates hepatic disfunction. If the client didn't take his morning dose of lactulose, he wouldn't have elevated ammonia levels and decreased level of consciousness this soon. These assessment findings don't indicate that the client is relaxed or avoiding the nurse.

Ammonia, the major etiologic factor in the development of encephalopathy, inhibits neurotransmission. Increased levels of ammonia are damaging to the body. The largest source of ammonia is from: The digestion of dietary and blood proteins. Excessive diuresis and dehydration. Severe infections and high fevers. Excess potassium loss subsequent to prolonged use of diuretics.

The digestion of dietary and blood proteins. Explanation: Circumstances that increase serum ammonia levels tend to aggravate or precipitate hepatic encephalopathy. The largest source of ammonia is the enzymatic and bacterial digestion of dietary and blood proteins in the GI tract. Ammonia from these sources increases as a result of GI bleeding (i.e., bleeding esophageal varices, chronic GI bleeding), a high-protein diet, bacterial infection, or uremia.

A patient who has undergone liver transplantation is ready to be discharged home. The nurse is providing discharge teaching. Which topic will the nurse emphasize most related to discharge teaching? The patient will obtain measurement of drainage from the T-tube. The patient will exercise three times a week. The patient will take immunosuppressive agents as required. The patient will monitor for signs of liver dysfunction.

The patient will take immunosuppressive agents as required. Explanation: The patient is given written and verbal instructions about immunosuppressive agent doses and dosing schedules. The patient is also instructed on steps to follow to assure that an adequate supply of medication is available so that there is no chance of running out of the medication or skipping a dose. Failure to take medications as instructed may precipitate rejection. The nurse would not teach the patient to measure drainage from a T-tube as a patient wouldn't go home with a T-tube. The nurse may teach the patient about the need to exercise or what the signs of liver dysfunction are, but these are not as important as the immunosuppressive drug regimen.

A client with carcinoma of the head of the pancreas is scheduled for surgery. Which of the following should a nurse administer to the client before surgery? Potassium Vitamin K Vitamin B Oral bile acids

Vitamin K Explanation: Clients with carcinoma of the head of the pancreas typically require vitamin K before surgery to correct a prothrombin deficiency. Potassium would be given only if the client's serum potassium levels were low. Oral bile acids are not prescribed for a client with carcinoma of the head of the pancreas; they are given to dissolve gallstones. Vitamin B has no implications in the surgery.

The nurse is administering Cephulac (lactulose) to decrease the ammonia level in a patient who has hepatic encephalopathy. What should the nurse carefully monitor for that may indicate a medication overdose? Watery diarrhea Vomiting Ringing in the ears Asterixis

Watery diarrhea Explanation: The patient receiving lactulose is monitored closely for the development of watery diarrhea stools, because they indicate a medication overdose. Serum ammonia levels are closely monitored as well.

A client with chronic pancreatitis is treated for uncontrolled pain. Which complication does the nurse recognize is most common in the client with chronic pancreatitis? Weight loss Diarrhea Fatigue Hypertension

Weight loss Explanation: Weight loss is most common in the client with chronic pancreatitis due to decreased dietary intake secondary to anorexia or fear that eating will precipitate another attack. The other answer choices are not the most common complications related to chronic pancreatitis.

A client is admitted for suspected GI disease. Assessment data reveal muscle wasting, a decrease in chest and axillary hair, and increased bleeding tendency. The nurse suspects the client has: cirrhosis. peptic ulcer disease. appendicitis. cholelithiasis.

cirrhosis. Explanation: Muscle wasting, a decrease in chest and axillary hair, and increased bleeding tendencies are all symptoms of cirrhosis. The client may also have mild fever, edema, abdominal pain, and an enlarged liver. Clients with peptic ulcer disease complain of a dull, gnawing epigastric pain that's relieved by eating. Appendicitis is characterized by a periumbilical pain that moves to the right lower quadrant and rebound tenderness. Cholelithiasis is characterized by severe abdominal pain that presents several hours after a large meal.

A preoperative client scheduled to have an open cholecystectomy says to the nurse, "The doctor said that after surgery, I will have a tube in my nose that goes into my stomach. Why do I need that?" What most common reason for a client having a nasogastric tube in place after abdominal surgery should the nurse include in a response? decompression instillation gavage lavage

decompression Explanation: Negative pressure exerted through a tube inserted in the stomach removes secretions and gaseous substances from the stomach, preventing abdominal distention, nausea, and vomiting. Instillations in a nasogastric tube after surgery are done when necessary to promote patency; this is not the most common purpose of a nasogastric tube after surgery. Gavage is contraindicated after abdominal surgery until peristalsis returns. Lavage after surgery may be done to promote hemostasis in the presence of gastric bleeding, but this is not the most common purpose of a nasogastric tube after surgery.

The nurse cares for a client after a gastroscopy for which the client received sedation. The nurse should report which finding to the physician? loss of gag reflex minor throat pain drowsiness difficulty swallowing

difficulty swallowing Explanation: The nurse should report difficulty swallowing to the physician as this may be a sign of perforation. Loss of gag reflex, minor throat pain, and drowsiness are expected findings after a gastroscopy for which the client received sedation and therefore there is no need to report to the physician.

Cystic fibrosis, a genetic disorder characterized by pulmonary and pancreatic dysfunction, usually appears in young children but can also affect adults. If the pancreas was functioning correctly, where would the bile and pancreatic enzymes enter the GI system? duodenum jejunum ileum cecum

duodenum Explanation: The duodenum, which is approximately 10 inches long, is the first region of the small intestine and the site where bile and pancreatic enzymes enter.

The major carbohydrate that tissue cells use as fuel is chyme. proteins. glucose. fats

glucose. Explanation: Glucose is the major carbohydrate that tissue cells use as fuel. Proteins are a source of energy after they are broken down into amino acids and peptides. Chyme stays in the small intestine for 3 to 6 hours, allowing for continued breakdown and absorption of nutrients. Ingested fats become monoglycerides and fatty acids by the process of emulsification.

What is the recommended dietary treatment for a client with chronic cholecystitis? low-fat diet high-fiber diet low-residue diet low-protein diet

low-fat diet Explanation: The bile secreted from the gallbladder helps the body absorb and break down dietary fats. If the gallbladder is not functioning properly, then it will not secrete enough bile to help digest the dietary fat. This can lead to further complications; therefore, a diet low in fat can be used to prevent complications.

A nurse educator is providing an in-service to a group of nurses working on a medical floor that specializes in liver disorders. What is an important education topic regarding ingestion of medications? metabolism of medications need for increased drug dosages need for more frequently divided doses medications becoming ineffective in clients with liver disease

metabolism of medications Explanation: Careful evaluation of the client's response to drug therapy is important because the malfunctioning liver cannot metabolize many substances.

When bowel sounds are heard about every 15 seconds, the nurse would record that the bowel sounds are normal. hypoactive. sluggish. absent.

normal. Explanation: Normal bowel sounds are heard every 5 to 20 seconds. Hypoactive bowel sound is the description given to auscultation of one to two bowel sounds in 2 minutes. Sluggish is not a term a nurse would use to accurately describe bowel sounds. The nurse records that bowel sounds are absent when no sound is heard in 3 to 5 minutes.

A client who was recently diagnosed with carcinoma of the pancreas and is having a procedure in which the head of the pancreas is removed. In addition, the surgeon will remove the duodenum and stomach, redirecting the flow of secretions from the stomach, gallbladder, and pancreas into the middle section of the small intestine. What procedure is this client having performed? radical pancreatoduodenectomy cholecystojejunostomy total pancreatectomy distal pancreatectomy

radical pancreatoduodenectomy Explanation: Radical pancreatoduodenectomy involves removing the head of the pancreas, resecting the duodenum and stomach, and redirecting the flow of secretions from the stomach, gallbladder, and pancreas into the jejunum. Cholecystojejunostomy is a rerouting of the pancreatic and biliary drainage systems, which may be done to relieve obstructive jaundice. This measure is considered palliative only. A pancreatectomy is the surgical removal of the pancreas. A pancreatectomy may be total, in which case the entire organ is removed, usually along with the spleen, gallbladder, common bile duct, and portions of the small intestine and stomach. A distal pancreatectomy is a surgical procedure to remove the bottom half of the pancreas.

Which symptoms will a nurse observe most commonly in clients with pancreatitis? severe, radiating abdominal pain black, tarry stools and dark urine increased and painful urination increased appetite and weight gain

severe, radiating abdominal pain Explanation: The most common symptom in clients with pancreatitis is severe midabdominal to upper abdominal pain, radiating to both sides and straight to the back.

The nurse is assisting the physician in a percutaneous liver biopsy. In assisting with positioning, the nurse should assist the client into a: high Fowler's position. lithotomy position. dorsal recumbent position. supine position.

supine position. Explanation: The nurse is correct to instruct the client to assume the supine position. Also, the nurse places a rolled towel beneath the right lower ribs.

A client with status asthmaticus requires endotracheal intubation and mechanical ventilation. Twenty-four hours after intubation, the client is started on the insulin infusion protocol. The nurse must monitor the client's blood glucose levels hourly and watch for which early signs and symptoms associated with hypoglycemia? 1- Sweating, tremors, and tachycardia 2- Dry skin, bradycardia, and somnolence 3- Bradycardia, thirst, and anxiety 4- Polyuria, polydipsia, and polyphagia

1

A client's blood glucose level is 45 mg/dl. The nurse should be alert for which signs and symptoms? 1- Coma, anxiety, confusion, headache, and cool, moist skin 2- Kussmaul's respirations, dry skin, hypotension, and bradycardia 3- Polyuria, polydipsia, hypotension, and hypernatremia 4- Polyuria, polydipsia, polyphagia, and weight loss

1

A health care provider prescribes short-acting insulin for a patient, instructing the patient to take the insulin 20 to 30 minutes before a meal. The nurse explains to the patient that Humulin-R taken at 6:30 AM will reach peak effectiveness by: 1- 8:30 AM. 2- 10:30 AM. 3- 12:30 PM. 4- 2:30 PM.

1

A hospital patient has been ordered a sliding scale of Humulin R for the duration of her admission. The patient's medication administration record specifies the first administration time of the day at 08:00 and the nurse knows that breakfast trays typically arrive on the unit between 07:45 and 07:50. When should the nurse administer the patient's insulin? 1- 07:30 2- 07:45 3- 08:00 4- 08:15

1

A nurse educates a group of clients with diabetes mellitus on the prevention of diabetic nephropathy. Which of the following suggestions would be most important? 1- Control blood glucose levels. 2- Drink plenty of fluids. 3- Take the antidiabetic drugs regularly. 4- Eat a high-fiber diet.

1

A nurse is preparing to administer two types of insulin to a client with diabetes mellitus. What is the correct procedure for preparing this medication? 1- The short-acting insulin is withdrawn before the intermediate-acting insulin. 2- The intermediate-acting insulin is withdrawn before the short-acting insulin. 3- Different types of insulin are not to be mixed in the same syringe. 4- If administered immediately, there is no requirement for withdrawing one type of insulin before another.

1

A nurse is providing education to a client who is newly diagnosed with diabetes mellitus. What are classic symptoms associated with diabetes? 1- increased thirst, hunger, and urination 2- Increased weight loss, dehydration, and fatigue 3- Loss of appetite, increased urination, and dehydration 4- Increased weight gain, appetite, and thirst

1

A nurse is teaching a diabetic support group about the causes of type 1 diabetes. The teaching is determined to be effective when the group is able to attribute which factor as a cause of type 1 diabetes? 1- Presence of autoantibodies against islet cells 2- Obesity 3- Rare ketosis 4- Altered glucose metabolism

1

A physician orders blood glucose levels every 4 hours for a 4-year-old child with brittle type 1 diabetes. The parents are worried that drawing so much blood will traumatize their child. How can the nurse best reassure the parents? 1- "Your child will need less blood work as his glucose levels stabilize." 2- "Your child is young and will soon forget this experience." 3- "I'll see if the physician can reduce the number of blood draws." 4- "Our laboratory technicians use tiny needles and they're really good with children."

1

Insulin is secreted by which of the following types of cells? 1- Beta cells 2- Melanocytes 3- Neural cells 4- Basal cells

1

Lispro (Humalog) is an example of which type of insulin? 1- Rapid-acting 2- Intermediate-acting 3- Short-acting 4- Long-acting

1

The nurse is describing the action of insulin in the body to a client newly diagnosed with type 1 diabetes. Which of the following would the nurse explain as being the primary action? 1- It carries glucose into body cells. 2- It aids in the process of gluconeogenesis. 3- It stimulates the pancreatic beta cells. 4- It decreases the intestinal absorption of glucose.

1

The nurse is describing the action of insulin in the body to a client newly diagnosed with type 1 diabetes. Which of the following would the nurse explain as being the primary action? 1- It enhances the transport of glucose across the cell membrane. 2- It aids in the process of gluconeogenesis. 3- It stimulates the pancreatic beta cells. 4- It decreases the intestinal absorption of glucose.

1

The nurse is preparing to administer a patient's scheduled dose of Novolin 70/30. When administering this dose of insulin, the nurse should: 1- Ensure that the insulin is not given near a previous injection site 2- Aspirate before injecting the insulin into the patient's subcutaneous tissue 3- Massage the injection site gently for 10 to 15 seconds after administration 4- Use a 3 mL syringe with a 24 gauge,- to-inch needle

1

The nurse is teaching an older client how to self-administer insulin. Which of the following would be most helpful to the client who is having difficulty drawing up the correct dosage of insulin in the syringe? 1- Syringe magnifier 2- Insulin pen 3- Jet injector 4- Insulin pump

1

Which clinical characteristic is associated with type 2 diabetes (previously referred to as non-insulin-dependent diabetes mellitus)? 1- Blood glucose can be controlled through diet and exercise 2- Client is usually thin at diagnosis 3- Client is prone to ketosis 4- Clients demonstrate islet cell antibodies

1

Which combination of adverse effects should a nurse monitor for when administering IV insulin to a client with diabetic ketoacidosis? 1- Hypokalemia and hypoglycemia 2- Hypocalcemia and hyperkalemia 3- Hyperkalemia and hyperglycemia 4- Hypernatremia and hypercalcemia

1

A 60-year-old client comes to the ED reporting weakness, vision problems, increased thirst, increased urination, and frequent infections that do not seem to heal easily. The physician suspects that the client has diabetes. Which classic symptom should the nurse watch for to confirm the diagnosis of diabetes? 1- Numbness 2- Increased hunger 3- Fatigue 4- Dizziness

2

A client is admitted to the unit with diabetic ketoacidosis (DKA). Which insulin would the nurse expect to administer intravenously? 1- Glargine 2- Regular 3- NPH 4- Lente

2

A client with diabetes mellitus is receiving an oral antidiabetic agent. The nurse observes for which symptom when caring for this client? 1- Polyuria 2- Hypoglycemia 3- Blurred vision 4- Polydipsia

2

A client with type 1 diabetes asks the nurse about taking an oral antidiabetic agent. The nurse explains that these medications are effective only if the client: 1- prefers to take insulin orally. 2- has type 2 diabetes. 3- has type 1 diabetes. 4- is pregnant and has type 2 diabetes.

2

A client with type 1 diabetes reports waking up in the middle of the night feeling nervous and confused, with tremors, sweating, and a feeling of hunger. Morning fasting blood glucose readings have been 110 to 140 mg/dL. The client admits to exercising excessively and skipping meals over the past several weeks. Based on these symptoms, the nurse plans to instruct the client to 1- administer an increased dose of neutral protamine Hagedorn insulin in the evening. 2- check blood glucose at 3:00 a.m. 3- eat a complex carbohydrate snack in the evening before bed. 4- skip the evening neutral protamine Hagedorn insulin dose on days when exercising and skipping meals.

2

A client with type 2 diabetes has recently been prescribed acarbose, and the nurse is explaining how to take this medication. The teaching is determined to be effective based on which statement by the client? 1- "I will take this medication in the morning, 15 minutes before breakfast." 2- "I will take this medication in the morning, with my first bite of breakfast." 3- "This medication needs to be taken after the midday meal." 4- "It does not matter what time of day I take this medication."

2

A diabetes nurse is teaching an older adult patient who has been recently diagnosed with type 2 diabetes about the importance of foot care. To reduce this patient's risk of developing a diabetic foot ulcer, what practice should the nurse teach? 1- The correct technique for soaking the feet in Epsom salts each evening 2- Technique for daily inspection of all of the surfaces of both feet 3- The correct placement of pillows to prevent the feet from being placed in a dependent position 4- Exercises that can be used to promote circulation in the patient's feet

2

A nurse is preparing a client with type 1 diabetes for discharge. The client can care for himself; however, he's had a problem with unstable blood glucose levels in the past. Based on the client's history, he should be referred to which health care worker? 1- Home health nurse 2- Dietitian 3- Psychiatrist 4- Social worker

2

A nurse is teaching a client with diabetes mellitus about self-management of his condition. The nurse should instruct the client to administer 1 unit of insulin for every: 1- 10 g of carbohydrates. 2- 15 g of carbohydrates. 3- 20 g of carbohydrates. 4- 25 g of carbohydrates.

2

An elderly patient has come to the clinic with his daughter. The patient is a diabetic and is concerned about foot care. The nurse goes over foot care with the patient and his daughter as the nurse realizes that foot care is extremely important. Why would the nurse believe that foot care is so important to this patient? 1- An elderly patient with foot ulcers experiences severe foot pain due to diabetic polyneuropathy. 2- Avoiding the complications associated with foot ulcers may mean the difference between institutionalization and continued independent living. 3- Hypoglycemia is a dangerous situation, and it may lead to unsteadiness and falls. 4- Drugs that patients are required to take for their diabetic condition often decrease circulation to the lower extremities.

2

An obese Hispanic client, age 65, is diagnosed with type 2 diabetes. Which statement about diabetes mellitus is true? 1- Nearly two-thirds of clients with diabetes mellitus are older than age 60. 2- Diabetes mellitus is more common in Hispanics and Blacks than in Whites. 3- Type 2 diabetes mellitus is less common than type 1 diabetes mellitus. 4- Approximately one-half of the clients diagnosed with type 2 diabetes are obese.

2

During a class on exercise for clients with diabetes mellitus, a client asks the nurse educator how often to exercise. To meet the goals of planned exercise, the nurse educator should advise the client to exercise: 1- at least once per week. 2- at least three times per week. 3- at least five times per week. 4- every day.

2

The nurse is preparing a presentation for a group of adults at a local community center about diabetes. Which of the following would the nurse include as associated with type 2 diabetes? 1- Onset most common during adolescence 2- Insufficient insulin production 3- Less common than type 1 diabetes 4- Little relation to prediabetes

2

A patient newly diagnosed with type 2 diabetes is attending a nutrition class specifically designed for type 2 diabetics. What general guideline would it be important to teach the patients at this class? 1- A low fat food generally contains low sugar. 2- Protein should be exclusively from animal sources. 3- Calorie intake should support a reasonable body weight. 4- Dietary fat should be eliminated from the diet.

3

During a follow-up visit 3 months after a new diagnosis of type 2 diabetes, a client reports exercising and following a reduced-calorie diet. Assessment reveals that the client has only lost 1 pound and did not bring the glucose-monitoring record. Which value should the nurse measure? 1- Fasting blood glucose level 2- Glucose via a urine dipstick test 3- Glycosylated hemoglobin level 4- Glucose via an oral glucose tolerance test

3

The diabetic client asks the nurse why shoes and socks are removed at each office visit. Which assessment finding is most significant in determining the protocol for inspection of feet? 1- Autonomic neuropathy 2- Retinopathy 3- Sensory neuropathy 4- Nephropathy

3

The nurse has cared for four patients with type 1 diabetes over the past few shifts. Based upon components of managing diabetes, which patient will likely have the greatest success in maintaining tight glucose control of his or her type 1 diabetes? 1- A patient who skips breakfast when her morning glucose reading is greater than 220 mg/dL. 2- A patient who never deviates from his prescribed dose of insulin. 3- A patient who adheres to a meal plan and meal schedule. 4- A patient who eliminates carbohydrates from her daily intake.

3

The nurse is teaching a client about self-administration of insulin and about mixing regular and neutral protamine Hagedorn (NPH) insulin. Which information is important to include in the teaching plan? 1- If two different types of insulin are ordered, they need to be given in separate injections. 2- When mixing insulin, the NPH insulin is drawn up into the syringe first. 3- When mixing insulin, the regular insulin is drawn up into the syringe first. 4- There is no need to inject air into the bottle of insulin before withdrawing the insulin.

3

When referred to a podiatrist, a client newly diagnosed with diabetes mellitus asks, "Why do you need to check my feet when I'm having a problem with my blood sugar?" The nurse's most helpful response to this statement is: 1- "The physician wants to be sure your shoes fit properly so you won't develop pressure sores." 2- "The circulation in your feet can help us determine how severe your diabetes is." 3- "Diabetes can affect sensation in your feet and you can hurt yourself without realizing it." 4- "It's easier to get foot infections if you have diabetes."

3

Which intervention is essential when performing dressing changes on a client with a diabetic foot ulcer? 1- Applying a heating pad 2- Debriding the wound three times per day 3- Using sterile technique during the dressing change 4- Cleaning the wound with a povidone-iodine solution

3

Which statement is true regarding gestational diabetes? 1- It occurs in the majority of pregnancies. 2- Onset usually occurs in the first trimester. 3- A glucose challenge test should be performed between 24 to 28 weeks. 4- There is a low risk for perinatal complications.

3

Which type of insulin acts most quickly? 1- Regular 2- NPH 3- Lispro 4- Glargine

3

A nurse is assessing a client with Cushing's syndrome. Which observation should the nurse report to the physician immediately? A. Pitting edema of the legs B. An irregular apical pulse C. Dry mucous membranes D. Frequent urination

B. An irregular apical pulse RATIONALE Because Cushing's syndrome causes aldosterone overproduction, which increases urinary potassium loss, the disorder may lead to hypokalemia. Therefore, the nurse should immediately report signs and symptoms of hypokalemia, such as an irregular apical pulse, to the physician. Edema is an expected finding because aldosterone overproduction causes sodium and fluid retention. Dry mucous membranes and frequent urination signal dehydration, which isn't associated with Cushing's syndrome.

Which diagnostic test is done to determine a suspected pituitary tumor? A. Radiography of the abdomen B. Computed tomography C. Measuring blood hormone levels D. Radioimmunoassay

B. Computed tomography RATIONALE CT or magnetic resonance imaging is used to diagnose the presence and extent of pituitary tumors.

A patient with a diagnosis of syndrome of inappropriate antidiuretic hormone secretion (SIADH) is being cared for on a medical unit, and the nurse is writing an individualized care plan. The priority nursing diagnosis for a patient with this condition is what? A. Deficient fluid volume B. Excessive fluid volume C. Hypothermia D. Hyperthermia

B. Excessive fluid volume RATIONALE The priority nursing diagnosis for a patient with SIADH is excessive fluid volume, as the patient retains fluids and develops a sodium deficiency. Restricting fluid intake is a typical intervention for managing this syndrome. Temperature imbalances are not associated with SIADH, so hyperthermia and hypothermia are not priority nursing diagnoses.

Which disorder is characterized by a group of symptoms produced by an excess of free circulating cortisol from the adrenal cortex? A. Cushing syndrome B. Addison disease C. Graves disease D. Hashimoto disease

A. Cushing syndrome RATIONALE The client with Cushing syndrome demonstrates truncal obesity, moon face, acne, abdominal striae, and hypertension. Regardless of the cause, the normal feedback mechanisms that control the function of the adrenal cortex become ineffective, and the usual diurnal pattern of cortisol is lost. The signs and symptoms of Cushing syndrome are primarily a result of the oversecretion of glucocorticoids and androgens, although mineralocorticoid secretion also may be affected.

A patient with a diagnosis of syndrome of inappropriate antidiuretic hormone secretion (SIADH) is being cared for on a medical unit, and the nurse is writing an individualized care plan. The priority nursing diagnosis for a patient with this condition is what? A. Deficient fluid volume B. Excessive fluid volume C. Hypothermia D. Hyperthermia

B. Excessive fluid volume RATIONALE The priority nursing diagnosis for a patient with SIADH is excessive fluid volume, as the patient retains fluids and develops a sodium deficiency. Restricting fluid intake is a typical intervention for managing this syndrome. Temperature imbalances are not associated with SIADH, so hyperthermia and hypothermia are not priority nursing diagnoses.

The nurse is caring for a client with diabetes who developed hypoglycemia. What can the nurse administer to the client to raise the blood sugar level? A. Insulin B. Glucagon C. Cortisone D. Estrogen

B. Glucagon RATIONALE Glucagon, a hormone released by alpha islet cells, raises blood sugar levels by stimulating glycogenolysis, the breakdown of glycogen into glucose, in the liver. Insulin is released to lower the blood sugar levels. Cortisone and estrogen are not released from the pancreas.

A client complains of nervousness and palpitations. Upon assessing the patient's heart rate, the nurse notes a heart rate of 120 bpm. Which of the following endocrine disorders is associated with palpitations and increased heart rate? A. Hypothyroidism B. Hyperthyroidism C. SIADH D. Hypoparathyroidism

B. Hyperthyroidism RATIONALE Hyperthyroidism increases the heart rate and palpitations. Thyroid hormone increases the cardiac contractility, cardiac output, and heart rate.

A client with diabetes mellitus develops sinusitis and otitis media accompanied by a temperature of 100.8° F (38.2° C). What effect do these findings have on his need for insulin? 1- They have no effect. 2- They decrease the need for insulin. 3- They increase the need for insulin. 4- They cause wide fluctuations in the need for insulin.

3

A client with type 2 diabetes asks the nurse why he can't have a pancreatic transplant. Which of the following would the nurse include as a possible reason? 1- Increased risk for urologic complications 2- Need for exocrine enzymatic drainage 3- Underlying problem of insulin resistance 4- Need for lifelong immunosuppressive therapy

3

A nurse understands that a major concern with type 2 diabetes is: 1- Older age (> 60 years). 2- Obesity (>20% of IBW). 3- Insulin resistance. 4- Overactive insulin secretion.

3

A client is admitted with hyperosmolar hyperglycemic nonketotic syndrome (HHNS). Which laboratory finding should the nurse expect in this client? 1- Arterial pH 7.25 2- Plasma bicarbonate 12 mEq/L 3- Blood glucose level 1,100 mg/dl 4- Blood urea nitrogen (BUN) 15 mg/dl

3

A client with type 1 diabetes mellitus is being taught about self-injection of insulin. Which fact about site rotation should the nurse include in the teaching? 1- Avoid the abdomen because absorption there is irregular. 2- Choose a different site at random for each injection. 3- Rotate sites from area to area every other day. 4- Use all available injection sites within one area.

4

A newly admitted patient with type 1 diabetes asks the nurse what caused her diabetes. In response, the nurse is explaining to the patient the etiology of type 1 diabetes. Which of the following is the most appropriate explanation by the nurse? 1- "The tissues in your body are resistant to the action of insulin, making insulin less effective." 2- "An acidic substance forms when your liver breaks down fatty acids because of the lack of insulin in your body." 3- "The secretion of placental hormones is causing your body to be resistant to insulin." 4- "Destruction of special cells in the pancreas causes a decrease in insulin production, and the level of sugar in your bloodstream increases."

4

A nurse is providing dietary instructions to a client with hypoglycemia. To control hypoglycemic episodes, the nurse should recommend: 1- increasing saturated fat intake and fasting in the afternoon. 2- increasing intake of vitamins B and D and taking iron supplements. 3- eating a candy bar if light-headedness occurs. 4- consuming a low-carbohydrate, high-protein diet and avoiding fasting.

4

A nurse is teaching a client recovering from diabetic ketoacidosis (DKA) about management of "sick days." The client asks the nurse why it is important to monitor the urine for ketones. Which statement is the nurse's best response? 1- "Ketones are formed when insufficient insulin leads to cellular starvation. As cells rupture, they release these acids into the blood." 2- "When the body does not have enough insulin, hyperglycemia occurs. Excess glucose is broken down by the liver, causing acidic by-products to be released." 3- "Excess glucose in the blood is metabolized by the liver and turned into ketones, which are an acid." 4- "Ketones accumulate in the blood and urine when fat breaks down in the absence of insulin. Ketones signal an insulin deficiency that will cause the body to start breaking down stored fat for energy."

4

After recently being admitted to the emergency department with signs and symptoms of hyperglycemia, a 33-year-old man was subsequently diagnosed with diabetes. The patient's blood sugars have been stabilized, and the man has begun diabetes education with a nurse educator. When working with this patient, the nurse educator should first: 1- Teach the patient about the essential concepts of nutrition 2- Explain the various insulin delivery devices to the patient 3- Ask the patient what questions he currently has about diabetes 4- Ascertain the patient's readiness and willingness to learn

4

Which information should be included in the teaching plan for a client receiving glargine, which is "peakless" basal insulin? 1- Administer the total daily dosage in 2 doses. 2- Draw up the drug first, then add regular insulin. 3- Glargine is rapidly absorbed and has a fast onset of action. 4- Do not mix with other insulins.

4

The primary function of the thyroid gland includes which of the following? A. Control of cellular metabolic activity B. Facilitation of milk ejection C. Reabsorption of water D. Reduction of plasma level of calcium

A. Control of cellular metabolic activity RATIONALE The primary function of the thyroid hormone is to control cellular metabolic activity. Oxytocin facilitates milk ejection during lactation and increases the force of uterine contraction during labor and delivery. Antidiuretic hormone (ADH) release results in reabsorption of water into the bloodstream rather than excretion by the kidneys. Calcitonin reduces the plasma level of calcium by increasing its deposition in bone.

A nursing student asks the instructor why the pituitary gland is called the "master gland." What is the best response by the instructor? A. "It regulates the function of other endocrine glands." B. "It is the gland that is responsible for regulating the hypothalamus." C. "The gland does not have any other function other than to cause secretion of the growth hormones." D. "It regulates metabolism."

A. "It regulates the function of other endocrine glands." RATIONALE The pituitary gland is called the master gland because it regulates the function of other endocrine glands. The term is somewhat misleading, however, because the hypothalamus influences the pituitary gland. The gland has many other hormones that it secretes.

Which of the following hormones controls secretion of adrenal androgens? A. ACTH B. TSH C. Parathormone D. Calcitonin

A. ACTH RATIONALE ACTH controls the secretion of adrenal androgens. When secreted in normal amounts, the adrenal androgens appear to have little effect, but when secreted in excess, as in certain inborn enzyme deficiencies, masculinization may result. The secretion of T3 and T4 by the thyroid gland is controlled by TSH. Parathormone regulates calcium and phosphorous metabolism. Calcitonin reduces the plasma level of calcium by increasing its deposition in bone.

During physical examination of a client with a suspected endocrine disorder, the nurse assesses the body structures. The nurse gathers this data based on the understanding that it is an important aid in which of the following? A. Detecting evidence of hormone hypersecretion. B. Detecting information about possible tumor growth. C. Determining the presence or absence of testosterone levels. D. Determining the size of the organs and location.

A. Detecting evidence of hormone hypersecretion. RATIONALE The evaluation of body structures helps the nurse detect evidence of hypersecretion or hyposecretion of hormones. This helps in the assessment of findings that are unique to specific endocrine glands. Radiographs of the chest or abdomen are taken to detect tumors. Radiographs also determine the size of the organ and its location. Antidiuretic hormone (ADH) levels determine the presence or absence of ADH and testosterone levels.

An adult patient is experiencing a temporary decrease in serum levels of T3and T4. What physiological response is this state likely to result in? A. Increased release of TSH B. Increased resorption of T3 and T4 in the renal tubules C. Release of sequestered T3 and T4 by the spleen D. Compensatory release of T5 by the parathyroid

A. Increased release of TSH RATIONALE If the thyroid hormone concentration in the blood decreases, the release of TSH increases, which causes increased output of T3 and T4. Low levels of thyroid hormone are not resolved by the action of the kidneys. Thyroid hormone is not sequestered in the spleen, and T5 does not exist.

A client is suspected to have a pituitary tumor due to signs of diabetes insipidus. What initial test does the nurse help to prepare the client for? A. Magnetic resonance imaging (MRI) B. Radioactive iodine uptake test C. Radioimmunoassay D. A nuclear scan

A. Magnetic resonance imaging (MRI) RATIONALE A computed tomography (CT) or magnetic resonance imaging (MRI) scan is performed to detect a suspected pituitary tumor or to identify calcifications or tumors of the parathyroid glands. A radioactive iodine uptake test would be useful for a thyroid tumor. Radioimmunoassay determines the concentration of a substance in plasma.

A client has a decreased level of thyroid hormone being excreted. What will the feedback loop do to maintain the level of thyroid hormone required to maintain homeostatic stability? A. Stimulate more hormones using the negative feedback system B .Stimulate more hormones using the positive feedback system C. Produce a new hormone to try and regulate the thyroid function D. The feedback loop will be unable to perform in response to low levels of thyroid hormone.

A. Stimulate more hormones using the negative feedback system RATIONALE Feedback can be either negative or positive. Most hormones are secreted in response to negative feedback; a decrease in levels stimulates the releasing gland.

Parathyroid hormone (PTH) has which effects on the kidney? A. Stimulation of calcium reabsorption and phosphate excretion B. Stimulation of phosphate reabsorption and calcium excretion C. Increased absorption of vitamin D and excretion of vitamin E D. Increased absorption of vitamin E and excretion of vitamin D

A. Stimulation of calcium reabsorption and phosphate excretion RATIONALE PTH stimulates the kidneys to reabsorb calcium and excrete phosphate and converts vitamin D to its active form, 1,25-dihydroxyvitamin D. PTH doesn't have a role in the metabolism of vitamin E.

A nurse is assigned to care for a patient with increased parathormone secretion. Which of the following serum levels should the nurse monitor for this patient? A. Glucose B. Sodium C. Calcium D. Potassium

C. Calcium RATIONALE Increased secretion of parathormone results in bone resorption. Calcium is released into the blood, increasing serum levels.

A client is being screened for a thyroid disorder. The nurse would anticipate that the client would most likely undergo which test? A. TSH B. T4 C. T3 D. needle aspiration

A. TSH RATIONALE Serum TSH is the best screening test for thyroid disorders and helps differentiate between disorders of the thyroid gland itself and disorders of the pituitary and hypothalamus. High levels of serum TSH will indicate normal function or hypothyroidism, whereas low values indicate hyperthyroidism. This is an example of the negative feedback system. If the thyroid hormones (T3 and T4) are not secreted by the thyroid, TSH is increased to help stimulate the thyroid to produce these hormones. If T3 and T4 are excreted in large amounts, as occurs in hyperthyroidism, TSH is decreased. Also TSH is used to monitor thyroid replacement regimen. Needle aspiration biopsy can be done to evaluate thyroid cell structure.

The nurse is assisting with the preparation of a patient who will undergo a radioactive iodine uptake (RAIU) test. The nurse should understand that this patient is being assessed for dysfunction of the: A. Thyroid gland B. Adrenal cortex C. Adrenal medulla D. Anterior pituitary

A. Thyroid gland RATIONALE The rate of iodine uptake by thyroid gland increases in hyperthyroidism and decreases in hypothyroidism. It is unaffected by changes in the structure or function of the pituitary or adrenal glands.

The nursing educator is teaching a group of new graduates about Cushing's disease. What symptom would the educator identify as being characteristic of Cushing's disease? (Select all that apply) A. Truncal obesity B. Hypertension C. Muscle weakness D. "Moon" face

A. Truncal obesity B. Hypertension D. "Moon" face RATIONALE Patients with Cushing's syndrome demonstrate truncal obesity, "moon" face, acne, abdominal striae, and hypertension.

A nurse is performing an examination and notes that the client exhibits signs of exophthalmos. What has the nurse observed? A. abnormal bulging or protrusion of the eyes B. excessive hair growth C. enlarged thyroid gland D. changes in pigmentation

A. abnormal bulging or protrusion of the eyes RATIONALE When there is an increase in the volume of the tissue behind the eyes, the eyes will appear to bulge out of the face. Exophthalmos is a bulging of the eye anteriorly out of the orbit.

A nurse is reviewing the laboratory order for a client suspected of having an endocrine disorder. The lab slip includes obtaining cortisol levels. What is being tested? A. adrenal function B. thyroid function C. thymus function D. parathyroid function

A. adrenal function RATIONALE The adrenal cortex manufactures and secretes glucocorticoids, such as cortisol, which affect body metabolism, suppress inflammation, and help the body withstand stress.

A nurse is developing a teaching plan for a client diagnosed with hyperparathyroidism that explains this condition. When describing the underlying problem, the nurse would most likely include a discussion about which mineral? A. calcium B. sodium C. potassium D. magnesium

A. calcium RATIONALE Hyperparathyroidism is characterized by having excess parathormone (PTH), leading to a markedly increased level of serum calcium that can present as a potentially life-threatening situation. Sodium, potassium and magnesium are not involved. Reference:

Which diagnostic test is done to determine suspected pituitary tumor? A. computed tomography scan B. measurement of blood hormone levels C. radioimmunoassay D. radiographs of the abdomen

A. computed tomography scan RATIONALE A computed tomography or magnetic resonance imaging scan is done to detect a suspected pituitary tumor. Radiographs of the chest or abdomen are taken to detect tumors. Radiographs also determine the size of the organ and their location. Measuring blood hormone levels helps determine the functioning of endocrine glands. A radioimmunoassay determines the concentration of a substance in plasma.

A nurse explains the role of the ovaries. Which hormones would be included in that discussion? A. estrogen and progesterone B. estrogen and progestin C. testosterone and progesterone D. estrogen and testosterone

A. estrogen and progesterone RATIONALE The ovaries produce estrogen and progesterone. Progestin is a synthetic compound. Testosterone is involved with the development and maintenance of male secondary sex characteristics, such as facial hair and a deep voice.

A nurse is teaching a client with an endocrine disorder how the nervous system and endocrine system are linked. Which structure would the nurse identify as the link between the two systems. A. hypothalamus B. brain C. medulla oblongata D. pancreas

A. hypothalamus RATIONALE The hypothalamus is the link between the nervous system and the endocrine system. The hypothalamus controls the pituitary gland, which secretes hormones to influence the target glands through the action of the secreting hormones. The brain and medulla oblongata are part of the nervous system. The pancreas is an endocrine organ.

Although not designated as endocrine glands, several organs within the body secrete hormones as part of their normal function. Which organ secretes hormones involved in increasing blood pressure and volume and maturation of red blood cells? A. kidneys B. cardiac atria C. brain D. liver

A. kidneys RATIONALE The kidneys release renin, a hormone that initiates the production of angiotensin and aldosterone to increase blood pressure and blood volume. The kidneys also secrete erythropoietin, a substance that promotes the maturation of red blood cells.

A client with a history of hyperparathyroidism comes to the emergency department complaining of extreme muscle weakness, vomiting, and bone pain. The client is diagnosed with hypercalcemic crisis. When providing care, the nurse would most likely administer which intervention if ordered? Select all that apply. A. large volumes of IV fluids B. diuretics C. phosphate therapy D. calcium gluconate E. propylthiouracil

A. large volumes of IV fluids B. diuretics C. phosphate therapy RATIONALE Acute hypercalcemic crisis can occur with extreme elevation of serum calcium levels. Serum calcium levels of greater than 15 mg/dL (3.7 mmol/L) result in neurologic, cardiovascular, and renal symptoms that can be life-threatening. Treatment involves rehydration with large volumes of IV fluids to keep urine output above 100 mL/hr (normal saline expands volume and inhibits calcium resorption); diuretic agents (to promote renal excretion of excess calcium); and phosphate therapy (to correct hypophosphatemia and decrease serum calcium levels by promoting calcium deposition in bone and reducing the GI absorption of calcium). Calcium gluconate would be given if hypoparathyroidism was involved. Propylthiouracil is used to treat hyperthyroidism.

A client is undergoing diagnostics for an alteration in thyroid function. What physiologic function is affected by altered thyroid function? A. metabolic rate B. growth C. fluid/electrolyte balance D. sleep/wake cycles

A. metabolic rate RATIONALE The thyroid concentrates iodine from food and uses it to synthesize thyroxine (T4) and triiodothyronine (T3). These two hormones regulate the body's metabolic rate.

A nurse assesses a female client and suspects that the client may be experiencing an excess in adrenocortical hormones. Which assessment finding would support the nurse's suspicion? Select all that apply. A. moon face B. facial hair C. buffalo hump D. truncal obesity E. exophthalmos

A. moon face B. facial hair C. buffalo hump D. truncal obesity

A nurse is providing care to a client who is experiencing low blood glucose levels. The nurse understands that the body attempts to raise the level by secreting which hormone? A. insulin B. glucagon C. somatostatin D. aldosterone

B. glucagon RATIONALE The beta cells of the pancreas secrete insulin, which facilitates glucose transport into body cells, thus lowering the blood glucose levels when they are above normal. When blood glucose levels are low, alpha cells of the pancreas secrete the hormone glucagon. It promotes gluconeogenesis to raise the blood glucose level. Because its action is opposite to insulin, it may be termed a counter-regulatory hormone. The delta cells of the pancreas secrete somatostatin, which reduces the rate at which food is absorbed from the GI tract. Aldosterone is secreted by the adrenal cortex and is responsible for regulating sodium balance.

An elderly woman complaining of weight gain, depression, and lethargy is diagnosed with hypothyroidism, and thyroid replacement is prescribed. During initiation of thyroid replacement therapy for the patient, the priority assessment for the nurse is to evaluate which of the following? A. Mental status B. Nutritional status C. Cardiovascular function D. Bowel function

C. Cardiovascular function RATIONALE Hypothyroidism is associated with coronary-related diseases such as elevated serum cholesterol, atherosclerosis, and coronary artery disease (CAD). When thyroid hormone is administered, the myocardial oxygen demand increases without increasing the myocardial oxygen supply; therefore, patients should be monitored for cardiac complications such as chest pain and congestive heart failure. Patients may also be treated for angina or arrhythmias due to the release of catecholamine's that may be activated during thyroid replacement therapy.

A nurse is performing a physical examination on client suspected of having an endocrine disorder. Which assessment finding might be indicative of a problem with the thyroid gland? A. Sudden weight loss without dieting B. Dilated pupils C. Cold intolerance D. Diarrhea

C. Cold intolerance RATIONALE The thyroid releases hormones that regulate the body's metabolic rate. A client with a malfunctioning thyroid gland may experience weight gain, constipation, cold intolerance, and slowing of body functions. Dilation of the pupils would more likely be related to the adrenal medulla secreting epinephrine and norepinephrine.

A client is having chronic pain from arthritis. What type of hormone is released in response to the stress of this pain that suppresses inflammation and helps the body withstand stress? A. Testosterone B. Mineralocorticoids C. Glucocorticoids D. Estrogen

C. Glucocorticoids RATIONALE Glucocorticoids, such as cortisol, affect body metabolism, suppress inflammation, and help the body withstand stress. Mineralocorticoids, primarily aldosterone, maintain water and electrolyte balances. The androgenic hormones convert to testosterone and estrogens.

The nurse is teaching a patient about nutrition and knows that the body needs specific nutrients in order to function normally. What element is essential to thyroid function? A. Potassium B. Selenium C. Iodine D. Chlorine

C. Iodine RATIONALE Iodine is essential to the thyroid gland for synthesis of its hormones.

The nursing educator is teaching a group of new graduates about Addison's disease. What symptom would the educator identify as being characteristic of Addison's disease? A. Truncal obesity B. Hypertension C. Muscle weakness D. "Moon" face

C. Muscle weakness RATIONALE Patients with Addison's disease demonstrate muscular weakness, anorexia, gastrointestinal symptoms, fatigue, emaciation, dark pigmentation of the skin, and hypotension.

A client is scheduled for a diagnostic test to measure blood hormone levels. The nurse expects that this test will determine which of the following? A. The concentration of a substance in plasma B. Details about the size of the organ and its location C. The functioning of endocrine glands D. The client's blood sugar level

C. The functioning of endocrine glands RATIONALE Measuring blood hormone levels helps determine the functioning of endocrine glands. A radioimmunoassay determines the concentration of a substance in plasma. The measurement of blood hormone levels will not reveal a client's blood sugar level. Radiographs of the chest or abdomen determine the size of the organ and its location.

A client is being seen in the clinic to receive the results of the lab work to determine thyroid levels. The nurse observes the client's eyes appear to be bulging, and there is swelling around the eyes. What does the nurse know that the correct documentation of this finding is? A. Retinal detachment B. Periorbital swelling C. Bulging eyes D. Exophthalmos

D. Exophthalmos RATIONALE Exophthalmos is an abnormal bulging or protrusion of the eyes and periorbital swelling. These findings are not consistent with retinal detachment.

A 47-year-old woman presents to her primary care provider complaining of bone pain. Routine laboratory studies reveal a high serum calcium of 12.0 mg/dL and increased PTH levels. Which of the following is the most likely diagnosis? A. Graves disease B. Cushing disease C. Addison disease D. Hyperparathyroidism

D. Hyperparathyroidism RATIONALE Hyperparathyroidism is characterized by having excess parathormone (PTH), leading to a markedly increased level of serum calcium

A nurse explains to a client with thyroid disease that the thyroid gland normally produces: A. iodine and TSH B. TRH and TSH. C. TSH, T3, and calcitonin. D. T3, T4, and calcitonin.

D. T3, T4, and calcitonin. RATIONALE The thyroid gland normally produces thyroid hormone (T3 and T4) and calcitonin. The pituitary gland produces TSH to regulate the thyroid gland. The hypothalamus gland produces TRH to regulate the pituitary gland.

A nurse explains to a client with thyroid disease that the thyroid gland normally produces: A. iodine and thyroid-stimulating hormone (TSH). B. thyrotropin-releasing hormone (TRH) and TSH. C. TSH, triiodothyronine (T3), and calcitonin. D. T3, thyroxine (T4), and calcitonin.

D. T3, thyroxine (T4), and calcitonin. RATIONALE The thyroid gland normally produces thyroid hormone (T3 and T4) and calcitonin. The pituitary gland produces TSH to regulate the thyroid gland. The hypothalamus gland produces TRH to regulate the pituitary gland.

A client who is frightened of needles has been told that he will have to have an intravenous (IV) line inserted. The client's blood pressure and pulse rate increase, and the nurse observes the pupils dilating. What does the nurse recognize has occurred with this client? A. The client is developing an infection. B. The client is having a response to dehydration. C. The client is in a hypertensive crisis. D. The client is showing the fight-or-flight response.

D. The client is showing the fight-or-flight response. RATIONALE The adrenal medulla secretes epinephrine and norepinephrine. These two hormones are released in response to stress or threat to life. They facilitate what is referred to as the physiologic stress response, also known as the fight-or-flight response. Many organs respond to the release of epinephrine and norepinephrine. Responses include increased blood pressure and pulse rate, dilation of the pupils, constriction of blood vessels, bronchodilation, and decreased peristalsis. The client does not demonstrate the signs of infection, dehydration, or hypertensive crisis.

During a follow-up visit to the physician, a client with hyperparathyroidism asks the nurse to explain the physiology of the parathyroid glands. The nurse states that these glands produce parathyroid hormone (PTH). PTH maintains the balance between calcium and: A. sodium. B. potassium. C. magnesium. D. phosphorus.

D. phosphorus. RATIONALE PTH increases the serum calcium level and decreases the serum phosphate level. PTH doesn't affect sodium, potassium, or magnesium regulation.


Kaugnay na mga set ng pag-aaral

Psyc Learning Curve: The Origins of Psychology

View Set